Tải bản đầy đủ (.pdf) (90 trang)

Luyện giải các bộ đề trước kì thi tuyển sinh đại học ba miền bắc- trung - nam môn tiếng anh (bản mới tái bản, sửa chữa, bổ sung)-part 1

Bạn đang xem bản rút gọn của tài liệu. Xem và tải ngay bản đầy đủ của tài liệu tại đây (47.91 MB, 90 trang )

428
527G iC i MITNAPHA VIET NAM
iiuiiy
luiii
6ien soan sdch ud Ddo tgo chuyen
Anh Ngd
VFIRSTBOOK
NGO VAN
MINH
' MITNAPHA VIET NAM
Trung
tam
Bien soqn
sdch
od Ddo tgo
chuyen
Anh NgU
VFIRSTBOOK
NGO VAN MINH
BAC

TRUNG
II
ii
TIENG ANH
(Tdi
ban,
sita chCta
ua bo
sung)
>t


Dank cho
hoc
sinh
Idp 12 tuyen
thi
Dai Hoc
-
Cao Dang
*
Bien somtheonoi dung va(Unhhudngmde
thi
cmbgGDsiDT
Lai
NOIDAU
Quy
thay
CO
giao va cac em hoc sinh than men!
Ky
thi
tuyin
sinh vao cac truong dai hoc, cao dang la mot
thi'r
thach Ion doi voi
cac em hoc sinh. Hoa chung voi khong khi on luyen miet mai cua cac em tren khap
ca nuoc,
chiing
toi ngay dem nghien cuu ya bien
soan
nhmig trang

sach
tieng Anh
b6 ich. N6i com trong thai gian nay la cuon Luyen
gidi
dS trm'rc ky thi Hai hoc ha
miin Bdc - Trung - Nam Tieng Anh.
Dieu
CO
ban truoc tien la cac bo de trong cuon
sach
nay duoc bien
soan
bam sat
cau true de thi
tuy^n
sinh vao cac truong dai hoc, cao dang cua Bp Giao due va Dao
tao. Dong
thai,
cac cau hoi dugc bien
soan
dira tren trong tam
kien
thi'rc xuyen suot
chuong
trinh
cac em theo hoc a nha truang. Cuon
sach
khong nham muc
dich
on

luyen
de hoc sinh "trung tu" khi thi nhung mong nguyen rang se la nguoi ban chi
duong dan loi khong th^ thieu cho cac em biet di sau vao con duang
kien
thuc ma
minh
se gap lai trong de thi tuyen sinh chinh thuc cua Bo Giao due va Dao tao.
Hau
het noi dung cua cuon
sach
dugc chinh
soan
gia - thay giao NGO VAN
MINH
nhieu nam
ixng
dung vao chuong
trinh
luyen thi dai hoc va tren thuc te da g§t
hai
thanh cong qua ket qua thi cua cac em hoc sinh. Cac ban doc gia se nhan ra loi
viet
ciia
cuon
sach
nay dugc tac gia the hien phan Ian voi giong dan giang bai gan
gui
nhu- tren lap hoc.
Moi
cau hoi chung toi deu dien

giai
kien
thuc rat ro rang voi ngon ngu gian di,
de hieu. Muc
dich
ciia
dieu nay khong chi giup cho cac em tiep can va nam that
virng
ki^n thuc ma con huang cho cac em nhieu sir img dung trong cac truang hop
khac cua de tuyen sinh.
Cu6n
sach
khong chi giup cho cac em hoc sinh tir on luyen ma con la mot tai
lieu
rat
hiru
dung doi vdi quy thay c6 giao, cac bac phy huynh dang quan tam giup
con
em
minh
chu^n bi
kien
thuc d^ buoc vao ky thi tuyen sinh sap toi.
Du
chung toi da c6
ghng
rk nhieu trong qua
trinh
bien
soan

nhirng cuon
sach
at
han con nhieu thieu sot. Rat mong nhan dugc y
kien
dong gop tir quy ban doc than
quen va gan xa de cu6n
sach
dirge hoan thien han trong nhiJng Ian tai ban.
Nhd sach Khang Viet xin
trdn
trong gi&i
thieu
t&i Quy doc gid vd xin Idng
nghe moi y kien dong gop decuon sach
ngdy
cdng hay
horn,
bo ich han. Thu xin
gui ve:
Cty
TNHH
Mot
Thanh
Vien - Dich Vu Van Hoa
Khang
Vi?t.
71 Dinh
Tien
Hoang, P. Dakao, Quan 1,

TP.
HCM
Tel:
(08)
39115694
-
39111969
-
39111968
-
39105797
-
Fax:
(08)
39110880
Hoac
email: ; thay
giao
Ngo Van
Minh
-
Email:
- mitnapha.edu.vn
Xin
chan thanh cam on!
Cty
TNHH
MTV
DVVH
Khang

Vi^t
Phan
I:
PHUONGPHAPHOCVAIAMBAITHITRACNGHIEM
NHAN
THliC
VE
HiNH
THUC
TRAC
NGHIEM
l.am
bai theo phuang
phap
trac nghiem la hanh dong
ciia
y thi'rc tir duy nham
chpn ra mot phuo'ng an
diing
nhat trong so nhieu phuang an de nghi doi vai mot
cau hoi.
DAC
DIEM
CUA
PHUONG
THUC
TRAC
NGHIEM

Bao quat

kien
thi'rc

H
Hieu
sau ve van de
kien
thirc va van d^
ciia
cau hoi
H
Ife
H
Chii
dong trong tu duy
H
S SI s Kha nang phan
tich
va phan doan van de doc lap
H HI s H
M
T6c do '
Do
dcS, thi sinh khi lam bai thi trac nghiem cung phai van dung va thi hien cac
yeu
to tren mai c6 dugc ket qua tot.
NHAN
DIEN
BAN
CHAT

VA
CACH
GlAl
QUYET
CUAMdlVANDE
TRONGDE
THI
TUYEN
SINH DAI
HOC-CAO
DANG
VAN
DE 1
PHAT
AM
(PRONUNCIATION)
BAN
CHAT
CUA VAN DE
Loai
cau hoi nay
kiem
tra nang
lire
nhan biet va phan biet
each
phat am
ciia
nguven
am va phu am.

Ddc
thii cua moi cau hoi: Moi cau hoi thuang c6 bon phuang an A, B, C, va
D.
Moi phuang an la mot tir hoan thien c6 chira thanh phan phu dm
hoac
nguyen
dm
(CO
the) giong nhau. Nhung chi c6 MOT thanh phan phu dm
hoac
nguyen dm
trong
so bon phuang an ay c6
each
phat am
KHAC
voi ba sir
lira
chpn con lai.
Nguyen tdc ra cle thi: Nguai
soan
de thi bao gia cung dua ra HAI phuang an
trong
so BON phuang an cua mot cau hoi eo chira thanh phan phu dm
hoac
nguyen dm dugc phat am nhu nhau ma thi sinh c6 the de dang nhan ra. Vai hai
phuang an con lai, doi hoi thi sinh phai can than de chgn ra mot.
Cdch
i^iai guyel van de: • - ,
-

Loai
gap HAI phuang an chira phu dm
hoac
nguyen dm dugc phat am nhu
nhau.
Luyfit
gidi
de
tru&c
ki t/ii
DH
3
miin
BJc,
Trung,
Nam
Tieng
An/i
-
Ngd
Van
Minh
-
Tap
trung
thai
gian
va nang
lire
de nhan ra thanh phan phu dm

hoac
ngiiyen
dm cua mot
trong
hai tir nay c6 each phat am
giong
ho$c khac vol hai
phuong
an da dugc
LOAI
GAP.

Hay tham i<hao each phat am cua phu dm va nguyen dm
trong
phan tong quat
ngCr
phap chua d phan sau cua sach.
Chi'ms
minh:
Mark
the letter A. B. C or D on
your
answer sheet to
indicate
the
word
whose
underlined
part
is

pronounced
differently from that of the
rest
in each of the
following
questions.
1
A.
ivory
B.
writer
C.
final
D.
window
2
A.
cjieck
B. chemistry C.
change
D.
cheap
(Trich
DE
THI
TUYEN
SINH
CAO
DANG)
BAI

GAI
AP
DUNG
PHUONG
PHAP

1. A.
ivory
B.
writer
C.
final
D.
window
Ta
nhan ra nguyen am
{trong
hai phuong an
LOAI
GAP B.
writer
va C.
final
dugc phat am /al/. Nhu vay, con lai hai truong hgp A.
ivory
va D.
window.
Ta
de dang nhan ra nguyen am i
trong

D.
window
dugc phat am la
/I/.
NghTa la
nguyen
am i
trong
piiuong
an A.
ivory
chac
chan dugc phat am la /al/.
Vay
dap an dung
ciia
cau hoi so
1
la D.

2. A.
clieck
B. chemistry C. cjiange D. dieap
Ta
nhan ra ch
trong trong
hai
phirong
an
l.OAl

GAP A. dieck va D. cjieap
dugc phat am la /tf/. Vay. con lai hai tnrang hgp B. chemistry va C. diange. Ta
khong
gap kho khan de nhan ra cji
trong
C.
change
dugc phat am la /tf/.
NghTa
la thanh phan di chi'ra
trong
ba phuang an A, C, va D co each phat am
nhu
nhau. Vay dap an dung cua cau hoi so 2 la B (ch
trong
cjiemistry
dugc phat
am
la
/k/).
v •
VAN
DE
2
TRONG
AM
(STRESS)
BAN
CHAT
CUA VAN DE

l.oai
cau hoi nay
kiem
tra nang
lire
nhan biet vi tri nhan trong am cua tir.
Dcic
thii
ciia
moi cau hoi: Moi cau hoi
thirong
co bon
phirong
an A, B. C, va D.
Moi
phuang an la mot tir hoan
thien
va co chua vj tri thanh phan dugc doc nhan
manh
(trong
am).
Nhirng
chi co
MOT
tir
trong
bon tir
irng
vai bon
phirong

an co
chi'ra
vj tri thanh phan dirge doc nhan manh khac vai ba sir
lira
chon con lai.
Njiuven
tdc ra de thi:
Nguoi
soan d^ thi bao gia
ciing
dua ra
HA!
phuong an
i
trong
so BON phuong an cua mot cau hoi co chua thanh phan dugc phat am
Cty
TISHH
MTVDVVH
KItang
Vi^t
manh
nhu nhau ma thi sinh co the de dang nhan ra. Vai hai phuang an eon lai,
doi
hoi thi sinh phai can than de chgn ra mot.
Cdch
giai
quyet van de:
-
Dgc to cac phuang an de trg

gii'ip
viec tim ra vi tri dugc phat am nhan manh
cua
tir.
-
Loai
gap
HAI
phuong an chua vi
tri
thanh phan dirge phat am manh nhu nhau.
-
Tap
trung
thai
gian
va nang lue de nhan ra tir chua vi tri thanh phan dugc
phat am manh
giong
hoac
khac voi hai phuong an da dugc
LOAI
GAP.

Hay tham khao phuang phap tim vi tri
trgng
am
ciia
tir
trong

phan tong quat
ngu
phap
chiia
o phan sau
ciia
sach.
Chung
minh
Mark
the letter A. B. C or D on
your
answer sheet to
indicate
the
word
that
differs
from the
rest
in the
position
of the main
stress
in each of the
following
(fuestions.
1.
A. agreement B. elephant C.
mineral

D.
violent
2.
A. electrician B.
majority
C. appropriate D.
traditional
(Trich
DE
THI
TUYEN
SINH
DAI HOC
NAM
2012)
BAI
GAI
AP
DUNG
PHUONG
PHAP

1. A. agreement B. elephant C.
mineral
D.
violent
Ta
de dang nhan ra hai tir Ian
lirgt
chua

trong
B. elephant va D.
violent
ccS
thanh
phan dugc phat am Ion hon
(trgng
am) rai vao am vi thu nhat. Voi hai
phuong
an con lai A. agreement va C.
mineral,
sau khi dgc Ion hai tir nay len, ta
khiing
gap kho khan de nhan ra C.
mineral
co thanh phan dugc phat am Ion hon
(trgng
am) rai vao am vi thu nhat. Vay dap an
diing
ciia
cau hoi na\a A.

2. A. electrician B.
majority
C. appropriate D.
traditional
Ta
de dang nhan ra hai tu Ian
lugt
chua

trong
B.
majority
va D.
traditional
co
thanh
phan dugc phat am Ion hon
(trgng
am) rai vao am vi thu hai. Voi hai
phuong
an con lai A. electrician va C. appropriate, sau khi dgc Idn hai tir nay
len.
ta khong gap kho khan de nhan ra C. appropriate co thanh phan dugc phat
am
Ion hon
(trgng
am) roi vao am vi thu hai. Vay dap an
diing
cua cau hoi nay
la
A.
Thuc
vay: .; . •
1.
A/o'gri:mont/
/'elif(3)nt/
/'minarol
/ /'vaiabnt/
2.

A
/ilek'trijn
/
/ma'd^tJriti/
/g'proupriot/
/tra'difangi/
Luy?n
gidi
de
trir&c
M
thi
DH 3
mien
BJc,
Trung,
Nam Tiing Anh - Ngd
VHn
Minh
VAN
DE 3
CAU
HOI CO
NHIEU
SLT
LLTA
CHQN
(MULTIPLE
CHOICE)
BAN

CHAT
CIIA
VAN DE '""^
Loai
cau
lioi
nay tap
trung
xoay vao cac cum tir c6
djnh,
tir
vimg.
ngfr
phap.
LUAT
CHUNG: Cac thi sinh can doc cac phirong an tra loi truoc de xac
dinh
miic
tieu ciia cau hoi la hoi ve
ngfr
phap,
cum tir c6
dinh
hay tir
vyng.
Xac
djnh
hinh
thi'rc cua cac phirang an tra ioi da dung hay chira.
THONG LE: BSt ky luc nao giai de thi ma dac biet la d^ thi theo

hinh
thurc
trie
nghiem, thi sinh hion dirge
nhac
nha
PHAI
DOC DE THAT CAN
THAN.
Day la mot viec lam can thiet nhimg neu cac thi sinh cu tap
trung
CAN
THAN
qua thi v6
hinh
chung dieu nay se phan tac dung trong viec giai quyet theo
phuang
phap tr§c nghiem. Thai gian rat quan trong. Neu mot cau hoi, chung ta
keo dai them 2
hoac
3 giay thi vai 80 can hoi, ta se mat bao nhieu thai
gian?
Lieu
rang chiing ta c6 giai hoan thien de thi ngay khi thai gian theo quy
dinh
ciia
Bo Giao due va Dao tao ket thi'ic hay khong?
PI
NC.DOC
vol

THONG
LE:
Xet da>
thiV
tir cac chCr cai sau day: ABCDEFGHK. (1)
Ta lai xet: ABC D F G H K (2)
N6ii
CO
de bai yen cau: Hay chon ra mot chiT cai
I'rng
vai mot phuong an tra
loi
phii
hop nhat de dien vao cho trong cua (2).
A.
P B. R C. S D. E
Theo
(2) ta de dang nhan ra chu cai phu hop nhat cho cho trong la E, tuc la
phucmg
an D. Ta KHONG CAN phai doc tir A d^n K mai tim dugc chfr cai con
thidu
trong (2) la E. Fa chi can quan sat nhiTng yeu to
di'rng
ngay trirac
vimg
trong
CO
D C —> B ^ theo
tinh
logic

thir
tir ciia bang chir cai thi E la
lira
chon duy nhat dung. Hoac ta chi can quan sat cac yeu to theo ngay sau
viing
trong
CO
F—> G
—<•
H —• theo
tinh
logic
thi'r
tir cua bang chfr cai thi E la
lira
chon duy nhat dung.
Tom
lai chimg ta se mat
nhieu
thoi
gian
de phai doc tir A den K mai tim
du-gc chfr cai con thieu trong (2) la E. Thay vao do, chung ta chi can doc thanh
phan
di'rng
ngay triroc
hoac
ngay sau
vimg
trong la dirge. Trong mot so trucrng

hop chimg ta chi can doc
phoi
hop ca hai thanh phan theo ngay triroc va theo
ngay sau
viing
trong thi c6 the chon dap an de dang.
Pltuonii
pluip
lam hai nay
t/iuirng
dp
(lung
cho cac cau hoi ve
ngie
phap
hay cum tir co
dinh.
Chimg
minh:
i,

Litis
interesting to take a new hobby such as collecting
stamps
or
going
fishing.
A.
up B. in C. over D. on
Cty TNHHMTVDVVH Khang

Vi^t
Dinh
dang
cau hoi: Hoi ve cum tir c6
dinh.
Ta chi can quan sat thanh ph§n a
new
hohhy
(mol ihii
lieu
khien
mai). ta quyet
djnh
ngay phu-ang an A. up. Vi ta
chi
CO
take up: theo
duoi
mot so thich nao do
• 2. The bad weather
caused
serious
damage
to the crop. If only it warmer.
A.
were B. had
been
C. has
been
D. was

Djnh
dang
cau hoi: Hoi ve ngfr phap. Ta chi can quan sat thay If
only,
ta
LOAI
GAP hai phuong an C. has
been
D. was vi chiing khong ton tai trong dang
If
only. Con hai phirong an A. were va B. had
been,
nghTa la
diing
thi qua
khir
dan
hay qua
khir
hoan thanh. Ta nhan ra dgng tir
caused
la
hinh
thiirc qua
khir
dan.
dien ta sir viec dfi qua. Do vay ta de dang quydt
djnh
B. had
been

la dap an
duy
nhat
diing.
• 3. You should look up the meaning of new words in the dictionary
misuse them.
A.
so that not to B. so not to C. so that not D. so as not to
Djnh
dang
cau hoi: Hoi ve ngir phap. Ta loai ba phirang an A. so that not to
B. so not to, va C. so that not that de dang. Vi theo ngay sau so
that
la mot menh
de. la khong c6
iiinh
thirc nhir phirong an B. Do vay, ta chon D la phirong an
duy
nhat
diing.
Qua that theo sau so as to + Vnguyen
• 4 smoking is a
causative
factor of many
diseases,
there is no ban
on
tobacco
advertising.
A.

However B. Therefore C.
Although
D. In spite of 'Y
Dinh
dang
cau hoi: Hoi ve ngfr phap. Ta thay
smokin}i nuwv
diseases
la
mot
menh de. Tir day, ta loai phuong an D vi sau hi
spite
of + N. Hai phuo-ng an
A.
However va B. Therefore khong logic ve nghTa vai menh de. Do do, ta quyet
dinh
C. la dap an duy nhat
diing.
• 5.
Scientists
have a lot of
research
into renewable energy
sources.
A.
done B. solved C. carried D. made
Djnh
dang
cau hoi: Hoi ve cum tir c6
dinh.

Ta c6 do a
research:
thi/c
hieri
CHOC
ii^iieii ci'ni. Do do, ta chon ngay A. la dap an duy nhat
diing.
• 6. The restaurants on the island are expensive, so it"s
worth
a packed lunch.
A.
taken B. taking C. to take D. take
Djnh
dang
cau hoi: Hoi ve ngfr phap. Ta c6
//
'.v
worth
+ V-ing:
That
dang
de j
lam gi. Vay, ta chon B. taking la dap an duy nhat
diing.
ri •
(Trich
DE THI TUYEN SINH DAI HOC 2012)
Luy$n
gidi
Oe

trudc k\
DH
3 mijn
Bdc,
Trung,
ISam
TiJng
Anit
- Ngd Van Minh
VAN
DE 4
CAU
CAN
NGHIA
(CLOSEST
MEANING
SENTENCE)
BAN
CHAT
CUA VAN DE
Loai
cau hoi nay xoay vao ba van de chu yeu: Dien dat ctia cac dien ngir, cau
true
ngfr phap tiroTig
dong
va ket hoD gifra cau true ngfr phap va dien ngfr.
Hinh
thirc de ra: Co mot cau van hoan chinh da cho (cau goc). Ben
dirai
la

bon
cau dien dat ung vai bon phuong an tra lai. Trong do chi c6
MOT
phuang
an dien dat dung y nghTa va hop ngfr
phap
nhat doi vai cau goc.
VIEC
PHAI
LAM:
Doc cau goc that i<y de nam bat y nghia va hinh thfrc
ngfr phap cua no. Muc dich
ciia
viec nay la de giup chung ta xac djnh muc tieu
phai
di tim trong bon phirong an ben
dirai
la dien dat
ciia
tu/cum tir hay ngir
phap
va
LOAI
GAP
ciic
phirong an c6 hinh thi'rc dien dat hay ngir
phap
khong
tircmg
dong

Chirng
minh:

1: Hardly had I
arrived
at the station when the train left.
A.
The train
leaves
as soon as we arrive at the station.
B.
No
sooner
had I arrived at the station than the train
left.
C. I had arrived hardly at the station when the train
left.
D.
The train didn't leave early as we had expected.
Muc
tieu hu-o'ng den la Ngu phap. Cau goc dang sir dung cau true c6 djnh
Hardly
when :
vira
thi
Nam bat muc tieu: Khi c6 cau true
Hardly
when : viva thi , ta
lien
tuang ngay den cau true dong nghTa No sooner than : vim thi

Kicm
djnh: Qua that phirong an B la toi uu. Cac phuang an con lai nhu A
khong
dirt7c chon vi iianh dong dang a hien tai; phuang an C tir hardly dung sai
\
tri;
phuang an C dien dat sai y vai cau goc

2: Nobody can deny that he is a new hero.
A.
Everybody said him to be a new hero.
B.
it is a fact that he is a new hero.
C. He is known by a new hero.
D.
People
said that, he was a new hero.
Muc
tieu hu-6"ng den la dien dat tir ngir.
Nam bat muc tieu: Khong ai co the phu nhan rang anh ta Id mot nguai hiing
mai.
Kiem
djnh: B. It is a fact that he is a new hero: Sir that la anh ta Id mot
nguai liimg
nu'ri.
Cac phuong an khac nhir A. Everybody said him to be a new
hero
(diing
khong hop
thi):

C. He is known by a new hero (sai y vai cau goc); D.
Cty
TNHH
MTVDVVH
Khang
Vi^t
People
said that, he was a new hero (sai thi vai cau goc).

3: / didn
V
arrive
in time to save the hoy.
A.
The boy wasn't
saved
by me.
B.
Unless
1
was late, I wouldn't
save
the boy.
C. If
I
arrived in time.
1
would
save
the boy.

D.
Had
1
arrived in time,
1
could
have
saved
the boy. Ji> i
Muc
tieu hu'o'ng den la ngfr
phap
ket hop vai dien ngfr.
Nam bat muc tieu: Toi da khong den kip
liic
de
cini
thdng he.
Ki^m djnh: D. Had I arrived in time, I could
have
saved
the boy. Phuang an
nay da
diing
cau dieu kien
loai
III
de dien dat tron y cua cau goc. Cac phuong an
con
lai khong phu hop ve nghTa va hinh thurc ngu

phap
vai cau goc.
(Trich
Ngan
luing
de thi
thir
tuyen sinh cua
MITNAPHA
VIET
NAM)
VAN
DE 5
PHAT HIEN
LOI SAI
(ERROR IDENTIFICATION)
BAN
CHAT
CUA VAN DE
Loai
cau hoi nay tap trung vao hai
ITnh
vuc ngir
phap
va cum tu co djnh ma
chiT
yeu la ngu- phap.
Hinh
thi'rc: Trong moi can dua ra co bon (theo thuong le) thanh phan dirge
gach chan

irng
vai cac ky tu A, B, C, D. Trong do co mot phuang an khong phu
hgp vai can van da cho. Chung ta hay chi ra phuang an nay.
Cach
giai quyet: Hay doc cau van can than, sau do ket hgp
giira
cac thanh
phan dirge dua ra lam phuang an dap an va cac thanh phan con lai trong can.
Neu
xac dinh ngu
phap
da on thi hay tap trung vao cac dien ngu (co
djnh).
Chirng
minh:
/ '

I. He considered to buy a new
house
but he has already changed his
mind
A
B
because
he supported his two
daughters
who are studying at the university.
C
D
Giai

quyet: Sai hinh
tiiuc
ngfr phap. A. to buy la khong phu hgp trong cau
van
tren. Ta co consider +
V-ing.

2.
Please
contact us
it'you
are in
need
with
our
services
again. ^
ARC
D , \
Giai
quyet: Sai hinh thi'rc dien ngiJ. D. with la khong dung. Ta co in need of:
can,
CO
nhii can
(Trich
Ngan hang de thi thu tuy^n sinh
ciia
MITNAPHA
VIETNAM)
LMyfn

gidi
Oi
Iru6c
Id thi DH 3
mien BJc, Trung,
Sam
Tieng Anii
- Ng/i
Van
Minli
^
VAN DE 6
OPC HIEU (READING COMPREHENSHION)
MUC f)iCH
ClIA
OE THI:
Kic>m
tra
nang lire
cam
thii ngon ngfr ciia
cac
thi
sinh.
Co
hai
dang
dac thu:
Dien
ti'r van cho

trong
va
chon phirong
an
dien
dat c6
noi dung theo
sat \oi
doan
van da cho,
Vt'ri
tlana
dien
tir vao cho
trdna: Doan
van cho san c6 mot so cho bi
khuyet
tir/cuni
tir. Ben
diroi
c6 cac can hoi
iing
vol moi cho
trong.
Moi can hoi c6 bon
phirang
an tra loi
nhirng
chi c6 mot
phiiong

an tra Idi
duoc chap nhan.
Hay doc
bon phirong
an tra lai
trirac
de xac
dinh
de bai hoi ve
ngfr phap.
tir
virng.
cum
tir
CO
djnh
ha>
gioi
tir. la
cung
IKMI
doc
luot that nhanh
ca
doan
van
(khong
qua
BA phiit)
de nam bhi noi

dung
co ban
toan
bai. Khi lam bai, cac thi
sinh
nen
tranh
toi da
viec
doc lap lai
nhieu
ca
doan
van de tim ra cau tra loi cho mot cau
hoi
vi rat mat
thai gian
ma
viec
lam nay
that
sir
khong
can
thiel.
Chi'rng minh:
We send
our
children
to

school
to
prepare them
for the
time when they will
be
big
and
will have
to
work
for
themselves. They learn their
own
language
(I)
they will
be
able
to
tell others cleariv what they want
and
what they know
and
understand what
(2)
tell them. They learn foreign
(3)
in
order

to be
able
to benefit from what people
in
other countries written
and
said,
and to
make
people from other countries understand what they themselves mean.

1. A. so
that
B. now
that
C. so far D. so
long

2. A. the
others
B.
other
C.
others
D.
another

3. A.
cultures
B.

customs
C.
languages
D.
habits
Giai
quyet:
Cau 1 hoi vc
ngil phap.
Doc bon
phuong
an
bcMi
diroi,
ta
biet
dirge dieu
nav.
Quan
sat
theo ngav
sau cho
trong
la mot
nienh
de c6 sir
dung
dang
S +
modals

+
Vnguyen.
la de
dang chon
A. so
that
la
phirong
an toi iru vi
trong
cac
phirorng
an
dira
ra thi chi c6
.s(;
thaf
+ S +
moikils
)
Vtiguyen
de tao ra
menh
de
trang ngfr
chi
miic
dich.
Cau
2 hoi ve

ngfr phap.
fa
LOAI
GAP B.
other
va D.
another
vi
theo nhir
ta
biet
other
t A' t
another
+
(one/N)
+ V(s6 it). Ta
chon
C.
others
vi
others
(mot so
khac)
+ V(s6
nhieu). PhiroTig
an A. the
others
(so con lai),
khong phii

hop v6i y
cua doan
van.
Cau
3 hoi ve hop tir. Ta co
forehini^iicif^cs:
n^oai
ngil:
Dira
vao y
ciia
cau
thir
hai
trong doan van,
ta
quyet dinh dirge dieu
nay.
Cty
T!\HH MTV
DVVH Khang
Vi^l
Vtifi
dans
dien
dot cd noi
duns
theo
sat
vtri doan

van M cho
Nguyen
tac:
Phai
doc va nam bat y - noi
dung
cua cau hoi
truoc
luc doc
doan van.
Ky
nang doc doan van:
Tap
trung vao
hai ky
nang
DQC
LUOT
de lay
thong
tin
va
DOC
CAN
CANH
da lay y.
Cach
the
hien:
Sau khi doc va nam bat y - noi

dung
cua can
hiii xong.
doc
do
len
doan
van. neu
thay
co noi
dung lien quan
thi hay dua ra loi
giai
cho cau
hoi
a\
Khong
can
phai
doc het ca
doan
van roi moi suy
nghT chon
dap an.
Chung
minh:
Traditionailv
in
America, helping
the

poor
was a
matter
for
private charities
or
local
government. Arriving immigrants depended mainly
on
predecessors
from
their homeland
to
help
them
start
a new
life.
In the
late
19"' and
early
20'''
centuries,
several
European nations instituted public
-
welfare programs.
But
such

a
movement
was
slow
to
take hold
in the
United States because
the
rapid
pace
of
industrialization
and the
ready availability
of
farmland seemed
to
confirm
the
belief that anyone
who was
willing
to
work could find
a job.
Most
of the
programs started during
the

Depression
era
were temporary relief
measures,
but one of
the programs
-
Social Security
- has
become
an
American
institution. Paid
for by
deductions from
the
paychecks
of
working people. Social
Security
ensures
that
retired persons receive
a
modest monthly income
and
also
provides
unemployment insurance, disability insurance,
and

other assistance
to
those
who
need
it.
Social
Security pavments
to
retired persons
can
start
at age 62,
but many
wait
until
age 65. when
the
payments
are
slightly higher. Recently, there
has
been
concern
that
the
Social
Securit\
fund
mav not

have enough money
to
fulfill
its
obligations
in the
21" centurv. when
the
population
of
elderl> Americans
is
expected
to
increase dramacticallv. Policy makers have proposed various ways
to make
up the
anticipated deHcit.
but a
long
-
temi solution
is
still being debated.
In
the
years
since
Roosevelt, other American presidents have established
assistance

programs.
These
include Medicaid
and
Medicare; food stamps,
certificates
that
people
can use to
purchase
food;
and
public housing which
is
built
at
federal
expense
and made available
to
persons
on low
incomes.
Needy
Americans
can
also
turn
to
sources

other than
the
government
for
help.
A
broad spectrum
of
private charities
and
voluntary organizations
is
available.
Voluntecrism
is on the
rise
in the
United States, especially among retired persons.
|
It
is estimated
that
almost
50
percent
of
Americans over
age 18 do
volunteer work,
and nearly

75
percent
of
U.S.
households
contribute money
to
charity.
Luy?n
gidi
Ok
tru&c
kl
tht
DH
3
mien
BJc,
Trung,
Nam
TIeng
Anh
-
J\go
Van
Minli
Question 1: New immigrants to the U. S.
could
seek
help

from
A.
the US government agencies B. volunteer organizations
C.
the people who came earlier D.
only
charity organizations
Question 2: It took welfare programs a
long
time to gain a
foothold
in the U.S.
due to the fast
growth
of
A.
population B. urbanization ^
C.
modernization D.
industrialization
Question 3: The
word
"instituted"
in the
first
paragraph mostly
means
A.
'"enforced" B. "introduced" C. "executed" D. "studied"
Question 4: The Social Security program has become possible thanks to

A.
people's willingness to
work
B. enforcement laws
C.
deductions
from
wages D. donations
from
companies
Question 5:
Most
of the
public
assistance
programs after the
severe
economic crisis.
A.
did not become
institutionalized
B. did not
work
in institutions
C.
were introduced
into
institutions D. functioned
fruitfully
in institutions

Question 6: That Social Security payments
will
be a burden comes
from
the
concern
that
A.
the program discourages
working
people
B.
younger people do not want to
work
C.
elderly people ask for more money
D.
the number of
elderly
people is
growing
Question 7:
Persons
on low incomes can
access
public
housing through
A.
state
spending B. federal expenditure

C.
low rents D. donations
Question S: Americans on low incomes can
seek
help
from
A.
government agencies B. federal government
C.
non - government agencies D.
state
government
Question 9: Public
assistance
has become more and more popular due to the
A.
people's
growing
commitment to charity
B.
innovations in the tax system i
C.
young people's voluntarism
only
D.
volunteer organizations
Question 10: The
passage
mainly
discusses

A.
immigration
into
America
B.
public
assistance
in
America
C.
funding
agencies in
America
D. ways of
fund
- raising in
America
(Trich
de thi tuyen sinh cua Bp Giao due va Dao tao)
Cty
TNHHMTVDVVH
Khang Vi$t
Phtn
II:
_
GIOITHIEU
24 Oi
THI
THIJTUYIN
SINH

DAI HOC
OE
THI
THLTSO
01
Th&i
gian lam bai: 90 pit lit ^
Mark
the
letter
A, B, C or D on your answer sheet to indicate the word
that
differs from the rest in the position of the main stress in each of the following
questions. ' *'
Question!: A.
simplify
B. magnificent C. conclude D. physique
Question 2: A.
rescue
B. particular C. geography D. mistake
Question 3: A. lettuce B. desire C. prefer D. upstairs
Question 4: A. clemantary B. pessimistic C.
infinitive
D. representative
Question 5: A. influence B. liberate C. gratitude D. incredible
Mark
the
letter
A, B, C, or D on your answer sheet to indicate the correct
answer to each

of
the following questions.
Question 6: Mr. Lee retired, he had
worked
as a
Marketing
consultant
for
the
Biotech
company.
A.
Since B. By the time C. For D. Provided that
Question 7:
Avery
Johnson participating in the commercial
fair
held
in
Japan
last year.
A.
considered B. persuaded C. made D. refused
Question 8: As there were too many participants at the seminar, the
main
speaker
them
into
four
smaller groups for discussion.

A.
divided
B. enrolled C. submitted D. provided
Question 9: A: How are you doing?
B:
A.
I am
walking
along the street. B. Thanks for asking.
C.
I am
carrying
out the project. D. Great. And you?
Question 10: It is interesting to a business course after the conference.
A.
take up B. take over C. take after D. take in
Question 11: Our
sales
team twenty members who completed successfully
the basic requirements. • • *'
A.
releases
from
B. put aside C. abides by D. consists of
Question 12: Mr. Chen managed to open the door he had no the key.
A.
much to B.
because
C.
if

only
D. on behalf
of
Question 13: No sooner his retirement than he wasn't received approval
from
the board of directors.
A.
did John announce B. John had announced
C.
John announced D. had John announced . . v ^
Question 14: - "Can I have some more
soda?"
A
r
m
f
II11
B. Its my pleasure
C,
Don'i
mention it D. make
yourself
at home
Question 15: He wondered his son looked
like
after
living
aboard for a
long
time.

A.
which
B. how C. when D. what
Question 16:
Owing
to the heavy fog, the
flight
from
Tokyo
to New
York
will
be
A.
overdue B. unpunctual C. expired D. unscheduled
Question 17: We had no reason to disbelieve his account of what happened.
A.
not to refuse to take over something
B.
to barely
imply
C.
to disapprove of what happened
D.
not to believe that something is true
Question IS: Mr. Steve: Are there an\s available?
Shopkeeper:
A.
1
hc\t me two thousand dollars.

B.
Yes. Ihey are convenient.
C.
No. fhey were made in
Viet
Nam.
n.
Yes. let me check in the warehouse.
Question 19: Mr. Pennington: i can't
find
my hat anywhere."
Mrs.
Jane:
" "
A.
Its time you bought another one.
B.
Its on my
dresser.
C.
It suits _\ou. , '
1).
It's no use wearing the tie.
Question 20: - "Vm
going
to Cuba for business."
A.
Congratulations! B. Never mind.
C.
Let me check your

flight
number. D. I wish you a
good
trip.
Question 21: Despite the fact that every cigarette pack has a warning from
Surgeon General, those who
still
smoke are smoking more heavily.
A.
printing
B. printer C. printed D. to
print
Question 22: In 2006. the
United
States go\c:::mcnt May 11 to be
Endangered Species Day.
A.
declared B. suggested C. issued D. criticized
Question 23: fhe
youth
will
our
traditional
operations.
A.
carr> on B. result in C. run off
D.bring
about
Question 24: The policeman .the victims to a safe place as quickly as possible.
A.

evacuated B. a Iterate C. flashed D. exaggerated
Question 25: The
staff
prefers to fly than to use other models of transportation
because
it is much more
A.
significant
B. convenient C. contended D. excessive
Question 26: His webpage explaining the
process
of course selection together
with
consulting
services is to any participant interested in receiving help.
A.
touch B. description C. regularity D. available
Question 27: At the latest meeting of Business
Advisor^'
Council,
the
newly
elected president that any new business be applied to the laws.
A.
obligated B. compensate C. proposed D. satisfied
Question 28: Bod\e ideas or thoughts by certain actions.
either
intentionall_\r
unintentionally.
A.

transmits B. disposes C. registers D. decline
Question 29: The band launched a movement that redesigned the
specification
of
rock
music in this country for the rest of the
decade.
A.
effect \^. effective C.
effectively
D. effectiveness
Question 30: we had deicided to cancel the
flight
to Toronto last week.
A.
If B,
If
only
C. Ever when D. Should
Mark
the
letter
A, R, C or D on your answer sheet to show the underlined
part
that
needs correction in each
ofthe/ollowinf;
questions.
Question 31: Mr.
C

hen has done a great
contribution
to the
building
of the
A
B CD
countiAside.
Question 32:
Advancing
in computer technology are
allowing
you to reach
A
B C
any part in the
world
b}
clicking
a mouse.
D
Question 33: No sooner had the professor stopped the lesson when
bell
rang. ,
A
BCD
Question 34:
1
he singer has rarely
ne\er

give a more impressive
live
show
A
B c
than
the one tonight. . ,
D
Question 35: The number of mone\ that vou gave me last week was not my refund.
ABC
D
!^ark the
letter
A, B, C or D on your answer sheet to indicate the word or phrase
flat
is
CLOSET
in
meaninf-
to the underlined
part
in each of the followin}>
questions.
Question 36: Whatever
1
said he
didn't
get rid of
smoking.
A.

carry out B. give up C.
gooff
D. put off
luyeti
f;iaidc
tnroc ki nil t)H3 mikn
B&c,
ImHS, mM Img
AHH
-l\gO van
Minn
Question 37: The
vast
number ofnew
businesses
has
been
in
danger
of bani<cruptcy.
A.
insensitive B. increasingly C.
tremendous
D. cautious
Question 38: Since her
parents'
death, she has
lived
with
her uncle.

A.
died for B. cried for
C.
been
brought up D. in touch
with
Mark the
letter
A, B, C or D on your
answer
sheet
to
indicate
the
word
or
phrase
that
is
OPPOSITE
in
meaning
to underlined
part
in each of the
following
questions.
Question 39: Workers are always advised to be cautious when entering and
leaving
the

place
due to a
recent
fire.
A.
careless
B. unfriendly C. ambitious D. factual
Question 40: They couldn't continue the big project
because
of lack of workers.
A.
plenty B. property C.
series
D.
limitation
of
Mark the
letter
A, B, C, or D on your
answer
sheet
to
indicate
the
sentence
that
is
closet
in
meaning

to each
of
the
following
questions.
Question 41: I regretted nol having applied for the
interesting
Job.
A.
I wish I could apply for the interesting job.
B.
I should
have
applied for the interesting job.
C.
1
must
have
applied for the interestingjob.
D.
I apologized for having applied for the interestingjob.
Question 42:
"I
didn 7 steal your handbag, Mary, " Peter said.
A.
Peter
accused
Mary of having stolen his
handbag.
B.

Peter
admitted having stolen Mary's
handbag.
C.
Peter
said that he didn't steal Mary's
handbag.
D.
Peter
denied having stolenm Mary's
handbag.
Question 43:
The
farm provides the residents with
fruits
and vegetables.
A.
Fruits and
vegetables
are produced on the farm by the
residents.
B.
The
residents
are supplied
with
fruits and
vegetables
by the farm.
C. The

residents
take
advantage
of the farm to grow fruits and
vegetables.
D.
The
residents
are no longer to use the farm to grow fruits and
vegetables.
Question 44: It is over
twenty
years since we last got in touch with them.
A.
We
have
beer
getting in touch
with
them for over 20
years.
B.
We haven't gotten in touch
with
them for over 20
years.
C. We didn't use to get in touch
with
them for over 20
years.

D.
We can't help keeping getting in touch
with
them for over 20
years.
Question 45: "To me, we should go to a remote area on the holidays, " he said.
A.
He
suggested
going to a remote
area
on the holidays.
B.
He advised his co-workers to go to a remote
area
on the holidays.
C. He let other people go to a remote
area
on the holidays.
D.
He found it impossible to go to ? remote
area
on the holidays.
Lty II\HH
.MIV
DVVH
Kltangl^iet
Question 46: "I
am
sorry

for
not finishing the project by deadline. Mary, " he said.
A.
He
accused
Mary of not having finished the project by deadline.
B.
He congratulated Mary on having finished the project by deadline. ,
C. Mar\d rather he finished the project by deadline.
D.
He apologized to Mary for not having finished the project by deadline.
Question 47: L/nfortunate/y. I
Mas
unable to attend Mr.
Miller's
performance last
night
A.
Not attending Mr.
Miller's
performance last night
made
me happy.
B.
However busy was
I,
I attended Mr.
Miller's
performance last night.
C. Mr.

Miller's
performance last night was so boring that I didn't attend.
D. 1
wish
1
had attended Mr.
Miller's
performance last night.
Question 4fi: You should take a taxi so thai you won't be late
for
the meeting.
A.
Unless you
take
a
taxi,
you
will
be late for
tiie
meeting.
B.
Without a
taxi,
you could
have
gone
to the meeting on time.
C. Pro\d that you
take

a
taxi,
you
will
be late for the meeting.
I).
Seeing that you took a
taxi,
you were not late for the meeting.
Question 49: \'o longer does
tlic
man smoke.
A. 1
he man has
been
smoking. B. Phe man has a habit of smoking.
C. The man didn't
stop
smoking. D. The man quit smoking.
Question 50: The hurricane destroyed hundreds
of
buildings
in the city.
A.
The hurricane resulted in destruction of
hundreds
of buildings in the
city.
B.
Ihc hurricane resulted

from
destruction of
hundreds
of
building
in the
city.
C. Despite the
appearance
of the hurricane, hundred of buildings in the
city
haven't in
danger
of destruction.
D.
'Hie hurricane
appeared,
however,
hundreds
of buildings in the
city
have
been
destroyed.
Read the
following
passage
and
mark
the

letter
A, B, C or D on your
answer
sheet
to
indicate
the correct
answer
to each
of
the
questions
from
51 to 60.
THE BEATLES
In
the l')60s, fhe
Beatles
were probably the most famous pop group in the
whole
world.
Since then,
there
have
been
a
great
many groups that
have
achieved

enormous fame, so it is
perhaps
difllcult
now to imagine how sensational the
Beatles
were at that time. They were four boys
from
the north of England and
none
of them
had any training in music. The\d by performing and recording
songs
by black
Americans and they had
some
success
with
these
songs.
Then they started
writing
their
own
songs
and that was when they
became
really popular. The
Beatles
changed
pop music, rhcy were the

first
pop group to achieve
great
success
from
songs
they
had
wriiieii
themselves.
After
that it
became
common for groups and
singers
to write
their
own
.songs.
The
Beatles
did not
have
a lon^
career.
Their
firsthit
record was in
1963 and they split up in 1970. Theyrs|<#jpped,dei«gTl}vQ ll966
Luy^n

gidi di
trir&c
ki
tlii
DH 3 miin
Bdc,
Trung,
!Sam
Tieng
An/i
- l\go
Van
Minh
because
it
iiad
become too dangerous for them - their fans were so excited that they
surrounded them and
tried
to tatce their clothes as souvenirs! However, today
some
of
their
songs
remain as famous as they were when they
first
came
out. Throughout the
world
many people can sing part of a Beatles song

if you
ask them.
Question 51: The
passage
is
mainly
about
A.
How the Beatles
became
more successful than other groups
B.
Why the Beatles
split
up after 7
years
C.
The Beatles" fame and
success
D.
Many people's
ability
to sing a Beatles song
Question 52: The
four
boys of the Beatles
A.
came
from
the

same
family
B.
were at the
same
age
C.
came
from
a
town
in the north of England
D.
received good
training
in music
Question 53: The
word
sensational" is closest in meaning to
A.
notorious B. bad C. shocking D. popular
Question 54: The
first
songs
of the Beatles were
A.
written
by themselves B. broadcast on the radio
C.
paid a lot of money D.

written
by black Americans
Question 55: What is not true about the Beatles?
A.
The members had no
training
in music.
B.
They had a
long
stable
career.
C.
They
became
famous when they wrote their own songs.
D.
They were
afraid
of being hurt by fans.
Question 56: The Beatles stopped their
live
performances
because
: ' ,;f
A.
they had earned enough money
B.
they did not want to
work

with
each other
C.
they spent more time
writing
their own
songs
D.
they were
afraid
of being hurt by fans.
Question 57: The year 1970 was the time when
A.
they
split
up B. they changed pop music
C.
they started their
career
D. they stopped doing
live
performances
Question
5H:
What the fans of the Beatles often did was
A.
sing together
with
them B. take their clothes as souvenirs
C.

ask them to
write
more
songs
D. ask them why they should
separate
Question 59: Some
songs
of the Beatles now
A.
are
still
famous as they used to be B.
became
too old to sing
C.
are sung by crazy fans D. are the most famous
Question 60: The tone of the
passage
is that of.
A.
admiration B.
criticism
C. neutral D.
sarcasm
Cly
TNHHMTVDVVHKItang
Vift
Read the following
passage

and mark the
letter
A, B, C or D on your answer
sheet
to indicate the correct answer to each
of
the questions from 61 to 70.
The Winterthur Museum is a
collection
and a house. There are many
museums devoted to the decorative
arts
and many
house
museums, but rarely in
the
United
States
is a great
collection
displayed in a great country house. Passing
through
successive
generations of a single
family,
Winterthur has been a private
estate
for more than a century. Even after the extensive renovations made to it
between 1929 and 1931, the
house

remained a
family
residence. This fact is of
importance to the atmosphere and effect of the museum. The impression of a
lived-in
house is apparent to the visitor: the rooms
look
as if they were vacated
only
a short
while
ago whether by the
original
owners of the furniture or the most
recent residents of the
house
can be a matter of personal interpretation. Winterthur
remains, then, a
house
in
which
a
collection
of furniture and architectural elements
has been assembled.
Like
an English country house, it is an organic structure; the
house, as
well
as the

collection
and manner of displaying it to the
visitor,
has
changed over the years. The
changes
have coincided
with
developing concepts of
the American arts, increased knowledge on the part of collectors and students, and
a progression toward the achievement of a historical effect in period-room displays.
The rooms at Winterthur have
followed
this current, yet
still
retained the character
of
a private house.
fhe
concept of a period room as a display technique has developed gradually
over the
years
in an
effort
to present works of art in a context that
would
show
them
to greater effect and
would

give them more meaning for the viewer.
Comparable to the habitat group in a natural history museum, the period room
represents
the decorative
arts
in a
lively
and interesting manner and provides an
opportunity
to
assemble
objects related by style, date, or place of manufacture.
Question 61: What
does
the
passage
mainly
discuss?
;'
A.
The reason that Winterthur was redesigned ,m. /ic •
B.
Elements that make Winterthur an unusual museum i it.
C.
How Winterthur compares to English country
houses
' '
D.
Historical
furniture contained in Winterthur

Question 62: The
phrase
"devoted to" in
line
1 is closest in meaning to
A.
surrounded by B. specializing " '* jn
C.
successful in D. sentimental about
Question 63: What happened at Winterthur between 1929 and 1931?
A.
The owners moved out B. The
house
was repaired
C.
The old furniture was replaced D. The
estate
became
a museum
Question 64: What
does
the author mean by stating "the impression of a lived-in
house is apparent to the visitor" (lines 5-6)?
A.
Winterthur is very old '''' |
B.
Few people
visit
Winterthur ' ''^
C.

Winterthur
does
not
look
like
a
typical
museum
D.
The
furniture
at
Winterthur
looks comfortable
Question 65: The
word
"assembled" in
line
9 is closest in meaning to
A.
summoned B.
,!•
'.'i
appreciated
C.
brought together D. fundamentally changed
Question 66: The
word
"it" in
line

9 refers to
A.
Winterthur
B.
},i,;<a
collection
C.
English
country house D.
visitor
no
ricril
vi J 'a
Question 67: The
word
"developing" in
line
1 1
is closest in meaning to
A.
traditional
B.
exhibiting
C.
informative
D.
evolving
Question 68:
According
to the

passage,
objects in a period
room
are related by all
of
the
following
RXCFPT
A.
date
B. style
C.
place of manufacture D.
past
ownership
Question 69: What is the relationship between the two paragraphs in
tiie
passage?
A.
Paragraph 2 explains a term that was mentioned in Paragraph i.
B.
F.ach paragraph describes a
different
approach to the display of objects in a
museimi.
C.
Paragraph 2 explains a philosophy of art appreciation that contrasts
with
that
explained

in paragraph i.
D.
Each paragraph describes a
different
historical
period.
Question 70: Where in the
passage
does
the author
explain
uhj displays at
Winterthur
have
changed?
A.
lines I 2 B. lines4-5 C. lines6-8 D. lines 10-12
Read
tlie
following'
passage
and mark the
letter
A, B, C or D on your answer
sheet to indicate the correct word for each
of
the blanks from 71 to 80.
According
to
airline

(71) statistics, almost 90 percent of
airline
accidents are survivable or
partially
unsurvivable. But
passengers
can increase their
chances
of (72) by learning and
following
certain
tips.
Experts say that
you
should read and
listen
to safet> (73) before
takeoff
and ask questions if
you
have uncertainties. You should (74) your
seat
belt low on your hips
and
as (75) as possible. Of course, you should also
know
how the
release
mechanism of \our belt operates.
During

takeoffs and landings, you are advised to
keep \i)ur feet
flat
on the
floor.
Before
takeoff
you should locate the
nearest
exit
and
an (76)
exit
and count the rows of
seats
between you and the exits so
that
you can
find
them in the dark
if
necessary.
In
the event that >ou are forewarned of the possible accident, you should put
your
hands
on your ankles and keep your head
down
until
the plane comes to a

(77)
stop. If smoke is present in the cabin, you should keep your head low
and
cover your face
with
napkins, towels, or
clothing.
If possible, wet
these
for
added (78) against smoke
inhalation.
To (79) as
quickly
as
possible,
follow
crew commands and do not take personal belongings
with
you. Do
not
jump
on
escape
slides before they are
fully
inflated,
and when you
jump,
do so

with
your arms and legs extended (80)
front
of
you.
When you get to the
ground,
you should move away
from
the plane as
quickly
as possible, and never
smoke
near
tiie
wreckage.
Question 71:
A.
industry
B.
section
C.
research
D.
field
Question 72:
A.
survive
B.
survival

C.
surviving
D.
survived
Question 73:
A.
direction
B.
observation
C.
instructions
D.
installation
Question 74:
A.
fasten
B.
enlarge
C.
endanger
D.
install
>
Question 75:
A.
tightly
B.
lighting
C.
tired

D.
heavily
Question 76:
A.
energic
B.
alternative
C.
changing
D.
magical
Question 77:
A.
complete
B.
completion
C.
completely
D.
completing
Question 78:
A.
protection
B.
prevention
C.
barning
D.
transportation
Question 79:

A.
evacuate
B.
disappear
C.
dismiss
D.
conflict*
,
Question 80:
A.
in
B.
on
C.
from
THE
END
DIEN
Gl\\P AN
Question 1 A.
simplif\/
B.
magnificent/mfeg'niflsnt/
C.
conclude
/lon'kluld/
D. physique
/fl'zilk/
Dap

an dung la A:
siniplif}'
c6 dau trong am danh vao am
tiet
thi'r
nhiit.
Cac ti'r
con
lai co dau trong am danh vao am
tiet
thi'r
hai.
Question 2 A. rescue/'reskju:/ B. particular/p3'tlkJ3b(r)/
C.
geography
/cl3i'Dgrnfi/
D. mistake
/mis'telk/
Dap
an
dimg
la A:
rescue
co dau trong am danh vao am
tiet
thu nhat. Cac tir con
lai
CO
dau trong am danh \ao am
tiet

thi'r
hai.
Question 3 A. lettuce/'letis/ B. desire/di'zals/
'at.i'i
.
C.
prefer
/pri'fDl/
D. upstairs /Ap'stesz/
Dap
an dung la A:
Icttiice
co dau trong am danh vao am
tiet
thi'r
nhat. Cac tir
con
lai co dau trong am danh vao am
tiet
thi'r
hai.
Question 4 A. elemantary
/eli'mentri/
B. pessimistic /pesi'mistik/
C.
infiniti\
/in'flnotiv/
D. representative
/reprl'zentotiv/
Dap

an dung la C:
in/initivc
co dau trong am danh vao am
tiet
thi'r
hai. Cac tir
con
lai co dau
Irong
am danh vao am
tiet
thi'r
ba.
Questions A, influence/'influsns/ B. liberate/'llbarelt/
C.
gratitude
/'grnetltjuld/
D. incredible
/in'kredabl/
Dap an dung la D: incredible c6 dau trong am danh vao am
ti6t
thu hai. Cac tir
con
lai c6 dau trong am danh vao am tiet thu nhat.
Question 6. Dap an dung la B. By the time
Giai
thich
ve mat ngu phap: Ta c6
By
the time +

clause
(qua
khi'r
dan),
clause
(qua
khir
hoan thanh)
Cac phuang an con lai khong phu hop: i /, A %>,
-
A. Since +
clause
(qua
khi'r
dan)/m6c
thai gian,
clause
(hien tai hoan thanh)
-
C. For + khoang thai gian,
clause
(hien tai nhoan thanh)
-
D. Provided that +
clause
(hien tai dcm),
clause
(tuong lai don)
Question 7. Dap dung la A. considered
Giai

thich
ve mat ngir phap:
consider +
V-ing:
xem xet viec gi
Cac phu-ang an con lai khong phu hop:
-
B.
persuade
+ s.o + V-to inf: thuyet phuc ai lam gi
-
C. make + s.o + Vnguyen: khien ai lam gi
-
D. refuse + V-to inf: tir ch6i lam gi
Question 8. Dap an dung la A.
divided.
Ta c6
I divide s.t
into:
chia cai gi thanh
Giai
thich
ve mat ngu nghTa: Vi c6 qua nhieu nguai tham gia vao buoi hoi thao
nen vi dien gia chinh da phan chia ho thanh bon nhom de thao luan cac van de.
Cac truong hop con lai khong phu hop:
-
B.
enroll
(v) dang ky tham gia
-

C. submit (v): nop, de
Irinh
-
D. provide s.o
with
s.t: cung cap cho ai cai gi
Question 9. Dap an dung la D. Great. And you?
Giai
thich
ve mat y nghTa: - Anh c6 khoe khong?

- Toi rat khoe. Con anh thi
sao?
Cac phirong an khac khong
phii
hop:
A.
I am
walking
along the
street:
loi dang di dao ngoai duang.
-
B. Thanks for asking: Cam on anh c6 lai hoi tham.
-
C.
1
am carrying out the project: loi dang thuc hien dir an nay.
Question 10. Dap an dung la A. take up: theo, theo du6i
Giai

thich
ve
matjigir
nghTa: That hay d^ chung ta theo hoc mot khoa
kinh
doanh sau hoi
nghi.
Cac phuong an con lai khong
phii
hgp:
-
B. take o\cr: cam quyen, dieu hanh
-
C. take after: giong
>
liet
-
D. take in: cho a tro
Question II. Dap an dung la P. consists of Ta c6
consist of: bao gom
niai
thich
ve mat v nahTa: D6i ban hang
ciia
chung toi gom c6 20 thanh
vien,
nhifng
ngirai
da qua huan luyen cac budc
kinh

doanh ca ban. , ^ ^ ,
Cac phirang an khac khong phu hop: . /, n*
A.
release
from:
thoat ra tir ,
.
B. put aside: gat
sang
mot ben
.
C.
abides
by: tuan theo ,fV nhi-bnrn ;n/« .
Question 12. Dap an dung la B.
because:
bai vi ^.^^
^^^.^,
r.iai
thich
ve mat
nuij
phap:
Because
+ clause: Bai vi
Cac phuang an con lai khong phu hop:
-
A. much to + N: mac dii
-
C.

if
only
+ clause: gia ma, irac gi
-
D. on behalf of + N: thay mat cho
Question 13. Dap an dung la D. had John announced
Giai
thich
ve mat ngu' phap:
No
sooner
+ had + S + V3/ed + than + S + V2/ed:
viia
moi thi.
Cac phirang an con lai khong
phii
hgp ve mat ngir phap.
Question 14. Dap an dung la D. make yourself at home: Xin ci'r tir nhien nhir d nha.
Giai
thich
ve mat
ngiT
nghTa: - Toi
dimg
them mot it
soda
nhe?
Cac phuang an con lai khong phu hgp y nghTa:
V/HV'
;

-
A. I'm
full:
Toi no roi.
-
C. Don't mention it:
Dirng
c6 ban tam ve dieu ay.
-
B. It's my pleasure: Do ia niem vui cua
toi.
"
Question 15. Dap an dung la I), what
Giai
thich
ve mat y nghTa: ()ng ta thSm nghT khong biet cau con
trai
cua
minh
trong
nhu the nao sau mot thai gian dai d nude ngoai.
Cac phuang an khac khong
phii
hgp.
Question 16. Dap an
dimg
la B. unpunctual: khong dung gid
Giai
thich
ve mat ngCr nghTa: Do c6 suong mil day dac nen chuyen bay tir Tokyo

den New
York
se den khong
diing
gid.
Cac phucrng an con lai khong
phii
hgp: * ' '
-
A. overdue: qua han 'i
-
C. expired: het han ni ; i ^
-
D. unscheduled: khong
CO
Ijch
trinh
Question 17. Dap an
dimg
la D. not to believe that something is true = disbelieve:
khong
tin tutmg
Cac phuang an con lai khong
phii
hgp:
-
A. not to refuse to take over something: khong tir chdi tiep quan van de gi
23
Luy^n
gmi

ue
irum
*i
ini un J
mien
Bar, i
nin<^,
mm i
wn);
nnn -1 vun
ivnnn
-
B. to barely
imply:
hau nhir khong c6 ngu y
-
C. to disapprove of what
happened:
khong tan thanh vai nhfrng gi da dien ra.
Question 18. Dap an dung la D. Yes. Let me check in the warehouse: Vang con a.
D6
toi k\km tra them trong nha kho.
Cac phuong an con lai khong
phii
hop:
-
A. They cost me two thousand dollars: Chung c6 gia hai ngan do la. ' ''^
-
B. Yes. They are convenient: Vang. Chung rat thuan tien. " '
-

C. No. They were
made
in Vietnam: Da khong. Chung dirge san xuat d
Viet
Nam.
Question 19. Dap an dung la B. It's on my
dresser:
No d tren ban trang diem cua em.
Giai
thich ve mat y nghTa: Anh khong tim thay cai mii dau ca.
Cac phirong an con lai khong phu hop:
-
A. it's time you bought another one: Da den luc anh phai mua mot cai moi. >
-
C. It suits you: No hop v6i anh day. fi " • ^
-
D. It's no use wearing the hat: That v6 ich khi doi cai mii nay.
Question 20. Dap an dung la D. I wish you a good
trip:
Clu'ic anh mot chuyen di
thanh cong.
Giai
thich ve mat ngCr nghTa: Toi chuan bi di den dat nirac Cuba.
Cac phirang an con lai khong
phii
hop:
A.
Congratulations! CluTc mimg anh!
B.
Never

mind:
Dung bao gio ban tam ve dieu ay.
C. Let me check your
flight
number: De toi kiem tra chuyen bay.
Question 21. Dap an dung la C. printed
Giai
thich ve mat y nghTa: Mac cho tren moi bao thuoc la deu ccS cau canh bao
ve sue khoe nhirng nhung nguoi hut thuoc la van hut ngay mot nang han.
Giai
thich ve ngCr phap:
Dimg
V3/ed nhu mot
tinh
tir mang y nghTa bi dong.
Cac tuong hop con lai khong
plui
hgp.
Question 22. Dap an dung la A. declared: cong bo
Cac phirong an con lai khong phu hgp y nghTa:
]^ ,^^^
^^.^^
-
B.
suggested:
de nghi
-
C. issued: ban bo, cap
-
D. criticized: phe binh

Question 23. Dap an dung la A. carry on: tiep tuc
Giai
thich ve mat y nghTa: The he tre se ke tuc cac phong trao truyen thong.
Cac phirong an con lai khong
phii
hgp:
-
B. result in: hau qua dim toi dau
-
C. run otT chay di. cha\n %
-
D. bring about: gay ra
Question 24. Dap an dimg la A.
evacuated:
so tan
Giai
thich ve mat y nghTa:
Vien
canh sat da so tan cac nan nhan den mot nai an
toan nhanh nhat ccS the.
Cac phirong an khac khong
phii
hgp:
B.
alterate: xen kc
C. flashed:
sang
len, chieu
sang
"*

I),
e iaggerated:
khuech dai i,*
Question 25. Dap an dimg la B. convenient: thuan tien
(;i|ii
jjljcii
ve mat y nghTa: Nhan vien nay thich di lai bang may bay hon cac
phirong
tien khac vi
tinh
thuan tien.
Cac phmrng an con lai khong
phii
hgp: - -
A.
significant: quan trgng, co y nghTa
C. contended: thi dau, tranh tai
D.
excessive:
virrt
qua
Question 26. Dap an
diing
la D. available
Jd
da b\k:
be available to s.o: san c6 cho ai
Giai
thich ve mat y nghTa: Trang web
ciia

ong ta
giai
thich qua
trinh
chgn
lira
khoa hoc cung nhir dira ra nhiTng loi tir van rat can thiet
ciio
tat ca cac ban hoc vien.
Cac phirong an khac khong
phii
hgp:
-
A. touch: chain \ao
-
B. description: sir mo ta
-
C. regularity:
tinh
deu dan
Question 27. Dap an dimg la C. proposed: de nghi
la
da biet:
SI
+
propose
that + S2 +
Vngiiyen
Cac phirang an con lai khong
phii

hgp:
-
A. obligated: thirc hien bon phan
-
B.
compensate:
den bii
-
D. satisfied: lam cho hai long
Question 28. Dap an dimg la A. transmits: truyen di
Giai
thich ve mat y nghTa: Ngon ngir hinh the truyen di y kien
hoac
suy nghT
ciia
chiing
ta bang hanh dgng dii do la c6 y hay khong c6 y.
Cac phirong an khac khong
phii
hgp:
-
B.
disposes
(oO:
giai
quyet. xir ly
"
C. regi.sters: dang ky tham gia
"
I), decline: tir choi, suy sup

Question 29. Dap an dimg la C. efTeclively (adv): mot
each
hieu qua
Qjai_thich
ve mat ngir phap: Ta can trang tir dung ngay
triioc
dgng tir tlurong.
Cac phirong an con lai khong
phii
hgp.
Question 30. Dap an dimg la B. If
only:
gia ma
,
Giajthich ve mat ngu phap: Chi c6
If
only mai dirge dimg
diiy
nhat mot menh de
^^tao day dii nghTa
ciia
mot can. Cac phirang an con lai can phai c6 hai menh de.
Question
31. Dap an dung la A.
done

made
Vi
ta c6:
make

a contribution to: cong hien cho cai gi
Question
32. Dap an dung la A. Advancing • Advances
Vi
ta c6:
Nl
+
giai
tir + N2 +
VQMl).
Quan sat cau goc, ta
tliay
are
dang
dirge dung.
Question
33. Dap an dung la C. when • than
Vi
ta c6:
No
sooner
.than.
Question
34. Dap an dung la A. rarely • khong diing
Vi
ta c6: Tir phu dinh never
dang
diroc dung trong cau. Va
rarely
(hiem

khi)
cimg
mang nghTa phu dinh.
1
lai tir phu dinh khong dung trong ciing mot cau.
Question
35. Dap an dung la A. number • amount [
Vi
ta c6:
the amount of + danh
tiir
khong dem dirge
Question
36. Dap an
diing
la B. give up = get rid of: tir bo
Cac phirang an
khac
khong
phii
hgp:
-
A. carry out: thirc hien
-
C. go off: bop CO
-
D. put off tri hoan
Question
37. Dap an
diing

la C.
tremendous
= vast: menh
mong,
khong 16, rat Ion
Question
38. Dap an
diing
la C.
been
brought up:
dugc
nuoi
duong
Cac phuong an con lai khong
phii
hgp:
-
A. died for: diet cho
-
B. cried for: rao
hang
-
D. in touch
with:
lien
he vdi
Question
39. Dap an dimg la A.
careless

* cautious: bit can ^ can than
Cac phuong an con lai khong phu hgp:
-
B. unfriendly: khong than thien
-
C. ambitious: tham vong
-
D. factual: sir that
Question
40. Dap an dimg la D.
limitation
of
=
lack of: thieu, hut
Cac dap an
khac
khong
phii
hcrp:
-
A. plenty: nhieu
-
B. property: tai san
-
C.
series:
hang
loat
Question
41. Dap an

diing
la B. Ly giai:
+
Ve mat v nghTa: Toi
l/igl
.sir
tiec vi eta khong nop ho sa I'tng
tuyen
vcio
cong
vice
iln'i
vi
ciy.
Hay noi
each
khac: Le ra toi nen nop ho scri'mg
tuyen
vao cong
viec
thu
vi
ciy.
Q
Danh gia: Hai dien ngfr "da khong" va "le ra" deu mang ban
ch§t
cua sir viec da
.xay ra trong qua
khir.
Ben canh do, chiing con the hien y hoi tiec

hoac
su trdch
moc. , , '
+
Ve mat ngir
phap:
Cau goc co cau
triic
Regret
+ (not) +
V-ing:
tiec vi da (khong) lam gi
y6'\g an B. cau true
chii
dao ciia no la:
Should +
have
+ V3/ed: le ra nen lam dieu gi
Ban
chat
ciia can true nay la the hien y hoi tiec
hoac
su trdch moc v6 dieu gi da
qua. Nhan dinh ve dieu nay. nen B la phuong an
dugc
lira
chgn.
o
Danh gia: ''"^ >
Regret

+ (not) +
V-ing
va Should + (not) +
have
+ V3/ed: tiec vi da
(khong) lam gi va le ra (khong) nen lam dieu gi
Chiing
ta c6 the
linh
hoat
diing
hai can
triic
nay de thay the cho nhau.
Cac phuong an
khac
khong
phii
hop:
Phirang an A
dang
diigc sir dung cau
triic:
.S'
+ wish + chaise (S + could/would + Vn^iiyen), the hien dieu iroc
muon dieu gi tra nen
khac
hon trong tirong lai
hoac
a hien tai

No
khong logic ve y nghTa voi cau goc.
Phirong an C
dang
diioc sir dung cau
triic:
Must
+ have +- V3/eci: At han da lam gi, the hien sir phan xet hay suy
doan (c6 ca so) ve dieu gi da xay ra trong qua
khir.
No
khong logic ve y nghTa voi cau goc.
Phirong an D mang can
triic:
apologize for st/V-ing: xin loi ve dieu gi/lam gi.
No
khong logic ve y nghTa voi cau goc.

MiV
rong:
1
wish I
iiad
applied for the interesting job.
If
only I had applied for the inlcresting job.
Question
42. Dap an
diing
la D. l.y

giai:
+
Ve mat v nghTa: "Toi da tdioni^ lay
Irgni
cdi
li'ii
xdch cua hgn. Mary g. " Peter
'loi.
Hay noi
each
khac
la:
Pelcr
dd phu niuiu viec lay Irgm cdi tui xdch cua Mary.
o
Danh gia: Cau goc
dang
iV
hinh thirc cau tuang thuat true tiep.
Hinh
thirc
phii
djnh
"da khong (didn't + Vngu>en
) "
dirge dien dat lai "denied +
V-ing :
phii
nhan". Tire la la dimg nipt dong tir mang y
iig/iia

phu dinh d^ thay the- cho
///////
'hi'fc
phii dinh cua mot dong tir.
^
Vgjnai
ngir
phap:
Ta can
nluV
,
,„.
, i ;
Cau trirc tiep c6 dang: didnT + Vnguycn

•T
Cau gian tiep: denied + having + V3/ed
I^an
chat
ciia hai cau
triic
tren la khang djnh dd
Idionii
/dm gi.
1 r;r
27
• Danli gia: Trong cau trirc tiep dung
clidn
7 +
Vn^iiyem

da khong lam thi ta c6
thO
dicn dat v tren trong cau gian tiep la
clcnifd
+
having
»
V3/ed:
choi
ho
lain
g'l
Hai cau true trC-n hoaii toan mang y nghTa nhu nhau.
la da biet:
deny
» V-i)i^:
phii
nhan
lain
^i: trong cau true tiep
diing
thi qua
khu
don, khi qua gian tiep ta chuyen thi qua khi'r dan thanh thi qua khi'r hoan
thanh. Do vay, didn't + Vnguyen (true tiep) se chuyen thanh denied + having +
V3/ed
(gian tiep).
Cac phircmg an
khac
khong

phii
hop:
- Phirang an A dang dirge sir dung can true:
accuse
s.o + of +
V-inf^:
khien trach/to cao ai lam gi.
No
khong logic ve y nghTa v6i cau goc.
Phirong an B dang dugc sir dung can true:
adniil
+
r-///^':
thira nhan lam gi.
No
khong logic ve y nghTa vai can goc.
- Phirong an C khong phu hop voi hinh thi'rc ngir phap. Khi cau tuang thuat true
tiep dung thi qua khiV don thi khi chuyen qua cau tirang thuat gian tiep ta phai dung
thi
qua khir hoan thanh.
• Mo long;
Peter
said to Mary he liad not stolen her handbag.
Question
43. Dap an dung la B. Ly giai:
+ Ve mat v nghTa: "\'6n<^
irai
ndv ciin,i^ cap ran qua cho dan cu- Ironi^ vitng." Hay
noi
each

khac
la: "Dan cir
irong
viing
diiyc
cung
cap rau c/ua lir
nong
Irai
ndyy
a Danh gia: Hai
each
dien dat: "Nong
trai
cung
cap
= dupe cung cap tu nong
trai".
Day chinh la hinh ihirc dien dat cua
each
ncSi
chii dong, bi dong.
+ Ve mat ngu phap: Can goc dung hinh thi'rc chu dong, voi cau true
provide
so
with
s.t: cung cap cho ai cai gi
Ta lai c6: supply s.o
with
s.t: cung cap cho ai cai gi

Qiian
sat ta tha\g an B dang dung hinh thi'rc bi dong ciia cau true
supply
s.o
Willi
S.I
de dien dat tron y nghTa ciia cau goc.
a Danh giaj '"" "
provide/supply s.o
with
s.t: cung cap cho ai cai gi
Cac phirang an
khac
khong phu hgp:
- Phmvng an A: Rau ci'i qua se dirge san xuat tren nong
trai
nay boi nhCrng nguai
dan. Thirc te, rau cii qua da vd
dang
dirge trong a nong
trai
(cau goc dung thi hien
tai
don). Do vay. phirong an nay khong logic ve mat y nghTa.
- Phuang an C: NhCrng nguai dan tan dung nong
trai
nay de trong cac loai rau
qua. Ta c6 cum tir:
Take
advantage

of s.t: Igi dung/ tan dung dieu gi (y tich cue)
No
khong logic ve mat y nghTa vai cau goc.
- Phirang an D: Nguai dan khong con sir dung nong
trai
nay de trong rau qua
nua. Ta da biet:
no longer: khong con nua
I
No
dugc
diing
trong hinh thi'rc cau khang djnh de chi vk sir
viec
hay hanh
dong nao do da tung xay ra va cham
dirt
trong qua khi'r. No khong logic y nghTa
vai
cau goc.
• Md rgng: Fruits and
vegetables
are pniduced on the farm by the
residents
=
Residents
use the
fann
to grow fruits and
vegetables.

Question
44. Dap an dung la B. Ly giai:
+ Vc mat
ntiu
nuhTa: DtJ
hern
hai
nnrai
nam
troi
qua ke sir Idn
SLUI
ciing
ohimg
'i'(>'i
lien
lac fx.'- Hay noi
each
khac:
Cliiing toi dd
khong
fi«w
4« wiipr.wig
.uidi
htm hai
mmri
nam qua.
a {)anh gia:
Khoang thai gian + ke tir Ian sau cung lam gi
= khong lam gi + khoang thai gian.

+ Ve mat ngu phap: Cau goc dang sir dung cau true:
It
is + khoang thai gian +
since
+
clause
(qua khir don): Da bao nhieu
thai
gian ke tir Ian sau cinig ai do lam gi)
Ta
CO
cau
triic
dong
y
nghia
voi no la:
S
+
has/have
+ not +
Vv,.d
+ for + khoang thai gian: ai do da khong
lam
gi trong khoang thai gian
• Danh gia:
'9^
S
+
has/have

+ not + V,
,,,|
+for + khoang thai gian
= It is (has
been)
+ khoang thai gian +
since
+ S +
last
+
Vi
cd
Cac phirong an
khac
khong phu hgp:
- Phuang an A:
Chung
toi
tiep
tuc
lien
lac vai ho
trong
h(rn hai
nuroi
nam qua.
No
khong logic ve y nghTa vol cau goc.
S +
has/have

+
been
+ Vjnj,
la hinh thi'rc cua thi hien tai hoan thanh tiep dien. Dien ta hanh dong
hoac
sir
viec
da .\ay ra trong qua khu keo dai tdi hien tai va nhan manh sir tiep tuc trong
tuang lai.
- Phuong an C: Toi
clnra
tinig
lien
he voi ho
trong
hon hai
imroi
nam
Inarc.
S + didn't + use ^- Vto inf: da chira tung lam gi
Y noi hanh dong
chua
tung xay ra truac day. No khong logic \di y nghTa cau goc.
Phuimg an D:
Chung
toi
khong
the
khong
lien

lac voi ho
trong
hai
mtnri
nam qua.
Can't + help +
V„,j,:
khong the khong lam gi
No
khong logic vai y nghTa ciia cau goc.
Luyftt
gidi
di
tm&c
kl thi DH 3
miin
Bdc,
Trung,
Nam Tiing
Anit
-
Ngd
Van
Minh
Question 45. Dap an la A. Ly
giai:
+
Ve mat v nghTa: ""Then toi, chung ta rten di den mot vimg ddt xa x6i
irorig
nhtmg

ngdy nghi tai^ anh ta noi. Hay noi
each
khae;
Anh ta de nghi
ciing
vai moi nguai
di
dc'n mot vitng ddt xa xdi trong nhimg ngdy nghi tai.
a
Danh gia: Cach dien dat Chung ta nen va anh ta de nghi vai moi nguai ,
dSu ham y nguofi noi
CO
tham gia hanh dong.
+
Ve mat ngu phap: Cau tucrng thuat true tiep su dung hinh
thue
We should +
Vbareinr:
chiing ta nen lam dieu gi
D6i
qua cau tuang thuat gian tiep: S (nguai noi) +
suggested
+
V-ing
Ban
chat
cua hai cau true tren deu ham y nguai noi eo tham gia hanh dong.

Danh gia:
We should + Vnpiyen

v(;n
S (nguoj noi)
sugget
+Vint.: Chung ta nen
Cac phuang an
khae
khong phu hop:
-
Phuang
an B: Anh ta khiiyen dong nghiep di den mot nai xa trong nhimg ngdy
nghi tai. No khong logic y nghTa vai cau goc vi nguai noi khong tham gia hanh
dong, ta
CO
cau true:
advise
s.o + Vto inf: khuyen ai lam gi
-
Phuang
an C: Anh ta cho phep nhimg ngia'ri khc'ic di den mot nai xa xdi trong
nhuvg ngdy nghi tai. No khong logic y nghTa vai cau goc. Ta eo cau true:
let
s.o +
Vhaaini:
cho
phep
ai lam gi
-
Phuang
an D: Anh ta nhgn
thdy

di den mot nai xa xdi trong nhimg ngdy nghi tai
Id
diet! khong the. No khong logic ve y nghTa vai cau goc. Ta c6 cong thuc:
S +
find
+ it + adj +
Vioini:
Ai do cam thay nhu the nao khi lam gi
Question 46. Dap an dung la D. Ly
giai:
+
Ve mat y nghTa: '"Toi xin loi vi da khong hodn thdnh du an dung thai hgn, Mary
a, " anh ta noi.
a
Danh gia: Cau goc dang a hinh thuc cau tuong thuat
trirc
tiep. Cau true
\.j
sorry For +
V-ing:
xin loi ve viec gi
Vi
dugc
dien dat lai
theo
hinh thCrc ngir
phap
khae
'
apologize to s.o for doing sth: xin loi ai do vi da lam gi

+
Ve mat mCr phap: Ta cSn nha
Cau true tiep c6 dang: S +
tobe
+ sorry + for +
V-ing
Cau gian tiep: S + apologize to s.o for doing sth
Ban
chat
cua hai cau true tren la dien ta sir xin loi ai do vi da lam gi.
Cae phuang an
khae
khong phu hap:
^ ^
,
-
Phuang
an A dang sir dung cau true
accuse
s.o of doing sth: khien trach, to cao ai lam gi.
Hinh
thuc ngu
phap
nay khong logic ve mat ngir nghTa vai cau goc.
Cty
TNHH
MTV
DVVH
KItang Vi,
-

Phirong an B dang
sur
dung cau true
j
congratulate s.o on
doing
sth:
chuc
mung ai ve viec gi.
Hinh
thurc nay khong
logic
ve mat ngu nghTa vai cau goc.
-
Phirang an C dang su dung hinh thuc ngir phap:
S| + would rather + Si +
WjkA-
Ngucri nao do mu6n ai lam gi hon.
Hinh
thuc ngir phap nay khong dien dat
logic
ve mat ngir nghTa vai cau g6e.
Question 47. Dap an dung la
D.
Ly giai:
+
Ve mat v nghTa: Khong may
thay,
toi khong the
tham

du buoi biiu diin
ciia
6ng
Miller toi qua.
Hay
noi each khae la: Ifac gi toi da
tham
du buoi bieu dien
ciia
ong
Miller
toi
qua.
+
Ve mat ngir phap: Cau goc dang sir dung thi qua khu don de noi den mot su
viec da dien ra trong qua
khiir.
Do do, ta phai sir dung cau dien dat vai
wish
khong
CO
that trong qua
khiir:
I
S + wish + clause (thi qua
khiir
hoan thanh)
Cac
phuang an con lai khong phu hop:
-

Phuang an A: Khong
tham
du buoi hieu
dien
ciia
ong
Miller
toi horn qua lam
toi
hanhphuc. Cau nay khong
logic
ve y nghTa vai eau goc.
-
Phuang an B: Mac dii ban nhimg toi da
tham
gia buoi bieu dien
ciia
ong
Miller
toi
qua. Cau nay khong
logic
ve y nghTa vai cau goc.
-
Phuang an C: Buoi hieu
dien
cua ong Miller toi qua qua chdn den noi toi du
khong
tham
du. Cau nay khong

logic
ve y nghTa vai eau goc.
Question 48. Dap an diing la A. Ly giai:
+
Ve mat y
nghTa:
Ban nen bat
taxi
de khong hi tre buoi hop.
Hay noi each khae la: Neu ban khong bat taxi thi hgn
.se
hi tre buoi hop.
+
Ve mat ngir phap: Cau goc dang diing hinh thurc cau dieu kien
loai
1 eo su dung
should + Vnguyen the. Dua vao y nghia ciia cau goc, ta cung c6 the
diing
eau
triic
cua
cau dieu kien loai 1 voi tir unless (Neu khong/Trir phi). Ta can luu y menh de
dung vai unices luon c6 hinh thirc khang dinh vi ban than unless da bao ham nghTa
phu dinh.
Cac phuang an con lai khong diing:
+
Phuang an B: Neu khong cd
taxi
thi ban c6 the di den buoi hop diing gia.
Without + N/N.P. S + could/would + have + V3/ed: N^u khong thi

Day la hinh thiirc dac
biet
cua cau dieu kien
loai
HI. Cau nay dien ta khong dung
ve mat y nghTa so vdi cau goc.
+
Phuang
an C: Mien la ban don
taxi
thi ban se tre buoi hop.
Provided that + clause (thi hien tai dan),
clause
(thi
tuomg
lai dcm): Mien la | ,n
LHrni
gmi
UL
limn
'jJilLLi^''
''""*•'
'
'""f>''
"""
' *"
"
"—
f)ay
cliinh

la hinh
tlu'rc
can 6\ki kien
loai
I vai cum tir Provided thai = If: Neu.
Cau van nay dien ta
kliong
di'ing
vc mat y nghTa so vai cau goc.
+
Plnrong an D: Boi ri han don la.xi iien ban
kfion}^
hi Ire hiidi hop.
Seeing
that +
clause, clause:
Bai vi
Cau na\n ta khong
diing
ve mat y nghTa so voi cau
JIQC.
Question 49. Dap an la D. I.y
giai:
+
Vc mat
V
nuhia: \^m)i dan
(m«ihmg£tmUeJihmx-mm
Hay
ncii

each
khac
Ji:
X^inn
djiK <imx'1'^^ ^
f^*^'-
\imm-oiiipbap:.
Ciu
p^'ic
ehu 4am ^vng
dang
thurc dao ngu cua trang tir
mo
Um^tT J
No
longer + tof) dong tir + S : khong con nixa.
Cau viet lai c6 th6 su dung mot trong cac cau true sau:
quit
* \'-inL;: tir bo vice gi.
uci
riti
of • \: tir bo
vice
gi
ui\
up t V-mg: tir bo viec gi
step

\'-ing:
tir bo viec gi

S • not + any more/any longer: khong con nira.
Cac dap an con lai khong
phii
hop:

I'lurong an .\ diiu ony: van chua ho hul thiuK. Thi hien tai hoan thanh
ticpilicn
o hinh (hue khaiiu dinh
<ii:
nhan manh vice hut thuoc cua
ngiroi
dan ong
na\n con Can n;i\n dat khong logic vc mat noi dung so \i cau goc.
1
Plnrong an B:
\^in'ri
dan oni^ c6 tlioi c/iwn hut
thmk.
[)icn dat khong logic ve
mat noi dung so \oi can goc.
+
Plnrong an C: S^iwn dan 6n\i da lihon^ tir ho viec
htit
thuoc. Dien dat khong
Igic
\ mat noi dung so
\(Vi
cau goc.
Question 50. Dap an
dung

la A. I.y
giai:
+
Vcimat y ng.hTa:
I'ran
hdo da
/>ha
hiiv
hany,
tram toil nhii Irony thiinh pho.
Ha\i
each
khac:
Iran
hdo dii dan den viec hiiny tram Ida nhd irony
thanh
phd
hi phd htiy.
Danh gia:
f)Oii
voi dong tir result, ta can kru y: '
+
result in: kcl (/iia diin den ddii
-t
result from: kel tpia lie ddu nid cd
Cac plnrong an con lai khong
dinig:
-
Plnrong an B: Tran hdo Id kel qua cua viec phd huy hiiny tram Ida nhd Irony ihiinh
pha la CO

cAu
triic
nlur da
dugc
noi o' tren: result from: kel qua do ddu ind cd.
-
Plnrong an C: Mac dii cd sir xual hien cua
Iran
hdo
nhimy
hiiny tram Ida nhd
irony lliduh phd van klidny ndin Irony
nyuy
ca hi tan phd. Dien dat khong logic ve
mat y nghTa so \6i can goc.
-
Phuxrng an D: Trdn hdo xudl hien. luy nhien. hiiny tram Ida nhd Irony Ihiinh
pho ltd hi idn phd. Dien dat khong
diing
y nghTa.
BAI
DICH
BE
NGHI
NHOM
NHAC
BEATLES
Vao
thap
nien 1960,

Beatles
dugc
biet den la mot nhom
nhac
theo
phong
each
dan gian hien dai noi
ti^ng
nhit tren toan the
giai.
Ke tir do, c6 rk nhieu nhom
nhac
noi
ti^ng
ra dai. Vi vay, c6 the noi chung ta kho hinh dung
dugc
sir anh
huang
cua
Beatles
vao thai diem do. Ho la bon
chang
trai
den tCr mien BSe
nuac
Anh va
khong ai trong so hg dirge dao tao chuyen nghiep ve am
nhac.
Ho khai nghiep

bang
each
trinh
dien va thu am nhirng bai hat cua nhom nguai My da den va hg da thanh
cong tir nhirng bai hat do. Sau do, ho bat dau tir
sang
tac
nhac
va chinh
giai
doan nay
la
luc hg that sir noi tieng.
Beatles
da lam thay doi dong
nhac
nhe duong dai. Ho la
nhom
nhac
dau tien thanh cong vai nhirng bai hat tir
sang
tac. Ve sau, viec tir
soan
nhac
de hat tra nen pho bien trong cac nhom
nhac
va ca sy. Nhom
Beatles
khong
theo

duoi
nghe
hat lau dai. Ban
nhac
noi tieng dau tien cua hg
dugc
thu am vao nam
1963 va nhom da tan ra vao nam 1970. Hg khong bi^u dien tryc tiep tir nam 1966 bai
vi
qua nguy hicm khi nhirng ngirai ham mg cuong nhiet cir c6 vay quanh ho d^ giat
lay
ao lam vat ky niem. Tuy nhien, ngay nay mot so bai hat cua nhom van con noi
tieng nhir hoi moi ra
miit.
Nhieu ngirai tren khSp thI
giai
c6 thI hat mot doan bai hat
do nhom
Beatles
sang
tac neu ban yen cau hg.
Question
51. C.
Chii
de chinh cua doan van la:
C.
Sir noi tieng va thanh cong ciia nhom
Beatles.
(No
dugc

de cap xuyen
suot
trong
bai van.)
A.
Nhom
Beatles
tra nen thanh cong hon nhirng nhom
khac
nhu the nao.
B.
Tai sao nhom lai
giai
tan sau 7 nam. - i
tsbt
D.
Nhieu nguai c6 the hat mot bai hat cua nhom
Beatles.
Question
52. C. Bon
chang
trai
nhom
Beatles
den tir mot thj tran a mien Bkc
nuac
Anh.
Trong bai c6 de cap a dong 4: "They were four hoys from the North of
Enyland
" (Hg la bon

chang
trai
d^n tir mien BSc
nuac
Anh )
A.
Cling
mot gia dinh.
B.
Cimg mot do
tu6i.
I
D.
Dugc dao tao tot ve am
nhac.
Question
53. C. Tir sensational c6 nghTa gan vai tir shocking: sir nao dong du-
doi
trong tam tri
A.
Tieng xau " >
BTe
D.
Noi tieng
Question
54. D. Nhung bai hat dau tien cua nhom
Beatles
dugc
sang
tac bai nhirng

nguai My da den.
-*^i*<d

Doan van c6 de cap a dong 5 va 6: "They started by performing and recording
•^ongs hy black Americans and
they
had some .success with the.se songs ". (Ho
khoi
•^Bhiep
bang
each
trinh
dien va thu am nhung bai hat cua nhom nguai My da den
hg da thanh cong vai nhirng bai hat do.)
LU}ifU
glU} uy
trUUL
m
llll
Ull J
mien
UUL,
ttuu^,
i
^um i
icfij;
/inn -1 ^gu r un imrnr
A.
Tir
sang

tac
B.
Phat
tren radio
C. Dugc tra nhieu tien
Question
55. B. Dieu khong dung ve nhom
Beatles
la ho c6 mot
nghe
on djnh
lau
dai.
Trong
bai c6 de cap a dong 10 va 11: "The Beatles did not have a long career ".
(Nhom
khong
theo
duoi
nghe
hat lau dai.)
A.
Cac thanh vien khong c6 sir dao tao ve am
nhac.
C. Ho tra nen noi tieng khi tir
sang
tac
nhac.
D.
Hp sg

ngiiy
hiem tir nh&ng nguai ham mo.
Question
56. D. Nhom
Beatles
khong bieu dien trirc tiep boi vi ho sg nguy hiem do
nhCrng nguai ham mg gay ra.
Dieu
nay
du-gc
de cap a dong 12, 13, 14 doan cuoi bai van: "They stopped
doing
lire
performances in 1966 because it had become too dangerous for
them
-
their fans were so excited
that
they
surrounded
them
and tried to lake their clothes
as souvenirs!" (Hg da khong bieu dien trirc tiep tu' nam 1966 boi vi qua nguy hiem
khi
nhifng nguoi ham mg qua cuong nhiet cir co vay quanh hg giat lay ao lam vat
ky
niem.)
A.
Hg da kiem du tien.
B.

Hg khong muon lam viec ciing nhau.
C. Hg danh nhieu thai gian han de
soan
nhac.
Question
57. A. Nam 1970 la nam nhom tan ra.
B.
Hg da lam thay doi nen
nhac
nhe
duang
dai.
C. Hg bat dau lap nghiep.
D.
Hg ngirng bieu dien true tiep tren san khau.
Question
5H. B. Viec ma ngirai ham mo cua nhom thirang lam la giat lay ao de lam
vat ky niem.
Dieu
nay dirge
nhac
den a euoi doan van: " their fans were so excited
that
they
surrounded
them
and tried to take their clothes as soin'enirs!" ( nhirng
ngirai
ham mg qua cuong nhiet ci'r co vay quanh hg giat lay ao de lam vat ky niem.)
A.

Hat ei'mg
vcVi
hg.
C. Yeu cau hg viet them nhieu bai hat. < >
D.
Hoi hg nguyen nhan nhom tan ra.
Question
59. A. Mot so bai hat cua nhom bay gia van eon noi tieng nhu ngay xua.
Doan cuoi bai van co de cap a dong 14, 15: "However,
today
some of their
songs remain as famous as
they
were when
they
first
came out". (Tuy nhien, ngay
nay mot vai bai hat cua hg van con noi tieng nhu hoi mai ra mat.)
B.
Da qua 16i thai d^ co the hat.
C. Dugc nhirng nguai ham mg cuong nhi^t hat.
D.
Noi tieng
nhat.
/ , , ,
Question
60. A. Diem nhan cua bai viet la sir ham mg.
B.
Sir phe binh
C. Sirtrung lap ,i

D.
Suchenhao
i'i.'^xv "ii\>mx\
BAI
DICH
DE
NGHI
u
Bao tang Winterthur chinh la mot bg suu tap va cung la mot ngoi nha. Co rSt
nhieu bao tang chi chuyen trung bay nhirng tac pham
nghe
thuat va nha co, nhung
hiem
co vien bao tang nao a
nuae
My co mot bg suu tap da
dang
duge
trung bay
trong mot ngoi nha rgng Ion d mien que. Trai qua nhiJng the he
lien
tiep nhau trong
mot gia dinh, Winterthur da tirng la tai san ca nhan trong han mot the ky. Tham chi
sau khi
dugc
nang
cap toan dien vao giira nam 1929 den nam 1931, ban
chat
cua
ngoi

nha van khong thay doi. Dieu nay co tac dong
tich
cue doi vai bau khong khi
va an tugng cua bao tang. Khach tham quan co the de
dang
nhan biet rang thirc
chat
day la mot ngoi nha co nguofi a: can phong trong ei'r nhu the no chi mai bj bo
trong
each
day mot thai gian ngan, nguai ta khong biet
lieu
nhirng ngudi chu ban
dau cua ngoi nha hay la nhirng ngudi chu gan day
nhat
mdi chinh la chu nhan that
sir. Tiep do, Winterthur van giCr lai
dugc
nguyen ven la can nha co bg suu tap tat ca
cac san pham trang tri ngi that va nhirng tac pham
nghe
thuat da
dugc
suu tam.
Tuong tu nhu mot ngoi nha nong thon d
nude
Anh, Winterthur co cku true rat
tinh
xao, can nha ciing nhu bg suu tap hay la
each

trung bay vdi du khach, da thay doi
Cling
vdi thai gian. Sir thay doi nay dien ra dong thdi vdi nhirng quan diem
nghe
thuat cua
nude
My
dang
dugc
hoan thien, no cung giup
nang
eao sir hieu biet cua
mot so thanh phan trong xa hoi bao gom cac nha suu tam
ciTing
nhu
gidi
sinh vien,
va
tien
trinh
hudng den nhirng thanh tuu ed tac dong mang
tinh
Ijch
su trong nhirng
cugc
trien lam
theo
tirng giai doan
phat
trien. Mac dau nhirng can phong tai vien

bao tang Winterthur da di
theo
xu hudng nay, no van con luu giu nhirng net dae
trung cua mot ngoi nha.
Y
tudng ve viec trung bay tac pham
theo
tirng giai doan
dugc
xem nhu la mot
each
thirc trung bay
dugc
hoan thien dan dan qua nam thang nhSm mue dich the
hien nhirng tac pham
nghe
thuat trong mot boi
canh
nhat
djnh ma ed the lam cho
khach tham quan thirc sir nhan ra gia tri han la chi biet qua y nghTa ben ngoai ciia
tac pham. Khac biet vdi nhirng bao tang
Ijch
sir tu nhien thi bao tang nay lai trung
bay cac tac pham
nghe
thuat doe dao
theo
mot
each

cue ky
song
dong va day Idi
cuon, dong thdi
giiip
ta hieu them nhirng gi
lien
quan den cd vat
chang
han nhu
kieu
dang, thdi gian va nai
ehung
xuat hien.
Question
61, B. Doan van chu yeu de cap den van de gi?
A.
Nguyen nhan ciia vi^c bao tang Winterthur
duge
thiet ke lai (khong
dugc
de
cap trong bai).
B.
Nhirng nhan to lam cho bao tang Winterthur trd nen doe dao (y nay xuat hien
xuyen
sudt
trong ca doan van).
C.
So vdi cac bao tang d mien que

nude
Anh thi Winterthur nhu the nao.
D.
Nhung do vat
Ijch
sir trong Winterthur (day chi la mgt y trong doan van).
Question 62. B. Cum tii devoted to a dong
1
c6 nghTa gSn x\hk vai:
A.
surrounded by: duoc bao quanh bai
B.
specializing in: chuyen ve cai gi, chuyen tarn ve van de gi
C.
successful in sth: thanh cong trong viec gi
D.
sentimental about sth/sb: da cam, nhay cam ve cai
gi/ai
Question 63. B.
Dieu
gi da xay ra a
Winterthur
giira
nam 1929 va 1931?
Ngoi
nha da duoc sua chiJa vi: trong doan 1
ciia
bai c6 de cap
"Even
after

the
extensive renovations
made
to it
between
1929 and 1931. the house remained a
family
residence
".
Cac dap an con lai khong
phii
hop:
A.
Nhfrng
ngiroi
chu nha da don di
C.
Nhiing
CO
vat da dirge thay the
D.
Ngoi
nha tro thanh
vien
bao tang
Question 64. C. Tac gia ngu y gi khi noi rang: "Khdch
tham
quan c6 the de dang
nhgn
hiet rang thirc chat day la mot ngoi nha co ngirai 6" (dong 5, 6)

Winterthur
khong
giong
nhir
nhOng bao tang khac: Doan 2 c6 de cap:
"Comparable to the
habitat
group in a natural history museum an
opportunity
to assemble objects related by
style,
date,
or place
of
manufacture
".
Cac dap an con lai khong
phii
hop:
A.
Winterthur
rat efi.
B.
It ngu'oi tham quan
Winterthur.
D.
Do \at trong
Winterthur
rat
tien

nghi.
Question 65. C. Tir "assembled" a dong 9 co nghla gan nhat vai
la
co:
assemble
(v):
ihu thgp. tu tap Igi = brought together
(v):
gom lai, hop lai
Cac dap an con lai khong
phii
hop:
A.
summon
(v):
trieu
tap
B.
appreciate
(v):
danh gia cao
D.
fundamentally changed: thay doi can ban
Question 66. A. Tir
"it"
a dong 9 de cap den:
Doan
i
CO
de cap: "Winterthur remains, then, a house country house, it is an

organic structure: the house, as well as the collection and manner
of
displaying it
to the
visitor,
has changed over the years. " ' ' ' ' '
Cac dap an con lai khong
phii
hop: ' ' ' ' '
B.
bp
siru
tap
C.
ngoi
nha a nong thon nuoc Anh
D.
khach tham quan
Question 67. D. Tir "developing" a dong 11 co nghTa gan nhat vai
Cae dap an con lai khong
phii
hgp:
A.
traditional
(a): truyen thong V | o
B.
exhibiting
(a):
tinh
chat pho bay

C.
informative
(a): co tac dung nang cao
kien
thurc, mang
tinh
thong tin
D.
evolving
(v):
hoan
thien,
phat
trien
Question 68. D.
Dira
vao doan van, nhCrng chu the
ciia
viec
trirng
bay cac tac phani
theo
tirng
giai
doan
lien
quan den tat ca cac yeu to sau ngoai
trir:
A.
date

(n):
thai
gian
B.
style (n):
kidu
dang W ,»
C.
place of manufacture: nai san xuat, nai xuat hien
D.
past
ownership: quyen sa
hiru
trong qua khur
Doan
2 co de cap: " arts in a
lively
and interesting manner and provides an
opportunity
to assemble objects related by
style,
date,
or place
of
manufacture.
Question 69. A. Moi quan he giOa 2 doan trong bai van nay la gi?
Doan
2 dien
giai
thuat ngO dugc de cap a doan I.

Ta
quan sat thay
cuoi
doan 1 co
nhae
den thuat
ngir
period-room: "a
progression
toward
the achievement of a historical effect in period-room
displays",
(tien
trinh
huang den
nhirng
thanh
tiru
co tac dong mang
tinh
lich
sir
trong
nhCrng cugc
trien
lam theo
tiing
giai
doan phat
trien.)

va dau doan 2
dicMi
giai
nghTa
cua thuat
ngfr
do: "The concept of a period room as a
display
technique has
developed
gradually over the years in an effort " (Quan
niem
\ viec
trirng
bay
tac pham theo
timg
giai
doan dirge xem
nhir
la mot each
thirc
trung bay dugc hoan
thien
dan dan qua nam thang nham muc
dich ).
B.
Moi doan mo ta mot each tiep can khac nhau doi v6i viec
trirng
bay cac tac

pham
nghe thuat trong
vien
bao tang.
C.
Doan 2 giang
giai
triet
ly
ciia
viec danh gia nghe thuat
trai
v6i quan
diem
a
doan 1.
D.
Moi doan
mieu
ta mot
giai
doan khac nhau
ciia
lich
sir. ' ' '
Question 70. D. Can hoi mang nghTa: Tdc gia giang gicii ly do viec truvg hay (f
vi^n hdo tang Winterthur
thay
doi a doan ndo?
-Dong

10-12.
Question 71. A. industry Question 72. B.
siirv
i\al
Question 73. C. instructions Question 74. A. fasten
Question 75. A.
tightly
Question 76. B. alternative
Question 77. A. complete Question 78. A. protection
Question 79. A. evacuate Question
HO.
A. in
Khai
thac tir
virng:
-
Airline
industrv': nganh hang khong
-
statistics: so
lieu
thong ke, nganh
thiing
ke
-
survivable: co the song sot
-
belt: day an toan
-
takeoffs: sir ha canh

-
installation:
sir lap dat vao
-
evacuate: sa
ciru
unsurvivable:
khong the song sot
operate: van hanh. boat dong
observation:
sir quan sat
instruction:
sir chi dan
conflict:
va chain, xung dot
DE THI
THU*
SO 02
Th&i
gian lam bai: 90 phut
Mark
the
letter
A, B, C, or D on your answer sheet to indicate the correct answer
to each
of
the following questions.
Question I: advertising is so widespread in the
United
States,

it has had an
enormous effect on American Mfe.
A.
Since B. Why C.
Because
of D. On account of
Question 2:
If
they are not careful
with
their accounts, their business
will
go
A.
poor B. bankrupt C. penniless D. broken
Question 3: The cat was to
wait
for the mouse to come out of its hole.
A.
patient enough B. so patient C. enough patient D. too patient
Question 4:
If
parents don't very
well,
their
children
will
be unhappy.
A.
get over B. get off C. get on D. get away

Question 5: The man who was taken to hospital had been for three hours.
A.
unfeeling B.
indifferent
C. unconscious D. sensible
Question 6: John never comes to
class
on time and
A.
neither doesn't peter , , B. neither
does
peter
C.
so
does
peter D. so doesn't peter
Question 7:
Hair
color is one of characteristics to be used in
identifying
people.
A.
the most obviously B. most obvious
C.
obviously the most D. the most obvious
Question 8: He
left
when he
failed
the exams the second

time.
A.
discouraged B. annoyed C. undecided D. determined
Question 9: of the workers has his own
work.
A.
Every B. Each C. Other D.
All
Question 10: Jim asked a good restaurant.
A.
if is this B.
if
this
is C.
if
was that D.
if
that was
Question II: 1 was very late. So, by the time I got to the station at 7.30, the 7.15
train
A.
left
B. has
left
C. was leaving D. had
left
Question 12: In the
United
States
is generally the responsibility of

municipal
government is always very good.
A.
for water treatment B. where water treatment
C.
which
water treatment D. water treatment
Question 13: John resigned as vice president in 1832 couldn't get along
with
President
Andrew
Jackson.
A.
because
of B. it was
because
C.
because
he D.
because
Question 14: I shall do the job to the
best
of my
A.
capacity B.
ability
C. knowledge D. talent
Question 15: "Where did you put your book? " - " "
A.
On the desk ,, , B. It costed me

five
dollars.
C.
It is useful D. By my father
Question 16: my
Japanese
friends
speak
English
so
you'll
have to
speak
Japanese.
A.
Both
of B. Neither of C.
All
of .j D. Some of
Question
// Traveling
by air is not cheap, and it isn't interesting
A.
either B. too C. neither D. also
Question IS:
If you
don't
mind,
we'd rather now.
A.

left
B. leave C. to leave D. leaving
Question 19:
IVfary
was the last applicant
A.
to be interviewed B. to be
interviewing
C.
to
interview
D. to have interviewed
Question 20: The numbers add to 70.
A.
off B. up C. in D. out
Question 21: The equipment in our
office
needs
A.
modern B. modernizing C. modernize D. modernization
Question 22: , the results couldn't be better.
A.
No matter what hard he
tried
B. No matter how hard he
tried
C.
Although
very hard he
tried

D. Despite how hard he
tried
Question 23: Kate: "Thanks for your help, Judy."
Judy:" "
A.
It's my pleasure B.
With
all my heart
C.
Never
remind
me D.
Wish
you a happy day
Question 24: This is my business and you have no
right
to
A.
intervene B. interfere C. interrupt D. breaking
Question 25: The
lift
is out of So,
we'll
have to
walk.
A.
function
B. order C. running D.
work
Mark

the
letter
A, B, C, or D on your answer sheet to indicate the sentence
that
is
closet in meaning to each
of
the following questions.
• '
' •
Question 26: We
haven'I
heardfrom
her for
over
ten
years.
r
A.
It's been over ten years since we last heard her.
B.
She said goodbye to us ten years ago.
C.
We last heard
from
her for ten years.
D.
It is over ten years since we last heard
from
her. , ^ ;

Question 27:
"Couldyou
show me the way to the
bookstore?"'
she said.
A.
She wants me to show her the way to the bookstore.
B.
She asked me if
I
could
show her the way to the book store.
C.
She
required
me to
show her the
way
to the book store.
D.
She
asked
me
if could
I
show her the
way to
the book store.
Question 28: She
tried

to
persuade
him lo get rid
of
smoking,
but she was
unsuccessful.
• <, *
A.
She
tried
in
vain to persuade him to get rid of
smoking.
,.i.j<ii4^c.
JI )
B. She
was
keen
on
persuading him
to
get rid of smoking.
C.
She
must
have
been successful
in
persuading to get rid

of
smoking.
D.
She
had difficulties
in
persuading him
to
get rid of smoking.
Question 29: I can do
everything
for you
because
1
believe
you.
A.
I
can do
everything for you
because
I
confide on you.
B. I
can do
everything fbr you
because
I
confide
in

you.
C.
There
is
nothing
I
can do
for you
because
I
confide
in
you.
D.
I
can do
everything for you
because
you
are
dependable.
*
Question 30:1
appliedfor
the
position
of a
secretary,
hut I haven't
received

any
replies
from
ihem.
A.
My
application for the position of a secretary
was
turned down.
B. I
was
refused
to
apply for the position
of
a secretary.
C.
I
will
be
dismissed from applying for the position of a secretary.
D.
I
gave
up
appl_\g for the position of a secretary.
V
Question 31: We
hare
run on/

of
rice.
A. There
is
no
rice.
B.
There isn't
any
rice.
C.
There
a
little
rice.
D.
There
is
no
more rice.
Question 32:
Everybody
has his own
good points.
A.
People
said that
he
has his
own good points.

B. It is
the fact that c\erybody
has his
own good points.
C.
People
said that
he
had
his
own good points.
D.
We
can't deny
he
had
his
own good points.
^,
Question 33: They
object
to
people laughing
at
their mistakes.
A. They object
to
laughing
at
their mistakes.

B.
They object
to
laugh
at
their mistakes.
^ ' ' ' '
C.
They object
to
being laughed
at
their mistakes.
^'
D. They object
to be
laughed
at
their mistakes.
' ' <*
Question 34:1
suggested you
go to
visit
him now.
A.
I
suggested going
to
\t him now.

B. You
must
have
gone
to
visit him.
C.
You
should
have
gone
to
\t him.
D. You
had
better
go to
visit him
now.
Question 35: Ms. Lan is promt
of
her
English.
A.
Ms Lan
prides on
her
F.nglisli.
B. Ms Lan
prides with

her
English.
C.
Ms Lan
boast
of
her English.
D. Ms
Lan's
English
is
admired.
40
Mark
the
letter
A, B, C or D on your answer sheet to indicate the word
that
differs from the rest in the position of the main stress in each of the following
questions.
Question 36: A.
lemonade
B.
magazine
C.
performance
D.
hospitality
^Mev//V>«
i7;

A.
mausoleum
B.
participate
C.
invitation
D.
malnutrition
Question 38: A.
intcresl
B.
manage
C.
harmonize
D.
machinery
Question 39: A.
mathematics
B.
personaiit\. recommend
D.
infrastructure
Question 40: A.
presence
B.
reality
C.
lecture
D.
newspaper

Mark
the
letter
A, B, C or D on your answer sheet to show the underlined
part
that
needs correction in each
of
the
followinf;
question.^.
Question 41:
1
found
it is
necessarv
for you
to
come here on time.
'""»
A
B CD
Question 42: It is
important that
she
complains about the problem
to Mr. John
A
B C D
Question 43: The

plane
to
Malavsia
was
late
10
minutes
so we .
A
B
couldn't
meet
you
as
early
as
possible.
D
Question 44:
Thev studied hardly
so
that they
can
pass
the examination easily.
ABC
D
Question 45:The
concert was so bored that ail the audience had gone home before
A

B C •
it
ended.
D
Question
46; Thank you \'er\ much
for
your letter whom
I
received
few
A
BCD
days
ago.
Question
47;
1
haven't heard from you when
you
left
for Da
Nang.
A
B C D
Question
48;
Unless
there
are no

taxis,
we
will walk
to
school.
A
B CD
Question
49; This
car
belongs with
my
brother.
He
bought
it m
2008.
A
B CD
Question
50;
We
are
looking forward
to
see
you
as
soon
as

possible.
A
B C D
iMvdH
SW ae
mriTV
KI mi UH J mm nuu, i mna, i wm i wn^ mm -;
^jju
» un mum -
Read the following passage and mark the letter A, B, C or D on your answer
sheet to indicate the correct answer to each of the questions from 51 to 60.
For more than six million American children, coming home after school means
coming back to an empty house. Some deal with the situation by watching TV.
Some may hide. But all of them have something in common. They spend part of
each day alone. They are called "latchkey children". They are children who look
after themselves while their parents work. And their bad condition has become a
subject of concern.
Lynette Long was once the pinciple of an elementary school. She said, "We
had a school rule against wearing jewelry. A lot of kids had chains around their
necks with keys attached. I was constantly telling them to put the keys inside the
shirts.
There were so many keys; it never came to my mind what they meant."
Slowly, she learned that they were house keys.
She and her husband began talking to the children who had keys. They learned
• of the effect working couples and single parents were having on their children. Fear
was the biggest problem faced by children at home alone. One in three latchkey
children the Longs talked to reported being frightened. Many had nightmares and
were worried about their own safety.
The most common was latchkey children deal with their fears is by hiding.
They may hide in a shower stall, under a bed or in a closet. The second is TV. They

often turn the volume up. It's hard to get statistics on latchkey children, the Longs
have learned. Most parents are slow to admit that they leave their children alone.
Question 51: The phrase "an empty house" in the passage mostly means
t
A. a house with nothing inside B. a house with no people inside
C. a house with too much space D. a house with no furniture
Question 52: One thing that the children in the passage share is that
A. they all watch TV B. they all wear jewelry
C. they spend part of each day alone D. they are from single - parent families
Question 53: The phrase "latchkey children" in the passage means children
' who
A. look after themselves while their parents are not at home
B.
close doors with keys and watch TV by themselves
, C. are locked inside houses with latches and keys
D.
like to carry latches and keys with them everywhere
Question 54: The main problem of latchkey children is that they
A. watch too much television during the day ,
B.
arc growing in numbers
, ;
. ' C. suffer a lot from being left alone
D.
are also found in middle - class families
Question 55: What is the main idea of the first paragraph?
V
A. Bad condition of latchkey children. B. Children's activities at home.
C. How kids spend free time. D. Why kids hate going home.
C/F

l[\HHimr
D VVH Kliang Viet
Question 56: Why did a lot of kids have chains around their necks with keys attacked?
A. Schools didn't allow them to wear jewelry, so they wore keys instead.
B.
They would use the keys to enter their houses when they came home.
C.
They were fully grown and had become independent.
nii
i
i
r
D.
They had to use the keys to open school doors.
Question 57: What do latchkey children suffer most from when they are at home
alone?
A. Fear B. Tiredness C. Loneliness D. Boredom
Question 58: Lynette Long learned to latchkey children's problems by
A. talking to them B. visiting their homes
C.
interviewing their parents D. delivering naires *
Question 59: What is the most common way for latchkey children to deal with
fears?
A. Talking to the Longs B. Hiding somewhere
C.
Lying under a TV D. Having a shower
Question 60: It's diflFicult to find out the number of latchkey children
because
A. they hide themselves in shower stalls or under beds
B.

they do not give information about themselves for safety reasons
C.
there are too many of them in the whole country
D.
most parents are reluctant to admit that they leave their children alone
Read the following passage and mark the letter A, B, C or D on your answer
sheet to indicate the correct answer to each of the questions from 61 to 70.
Very few people in the modem world obtain their food supply by hunting and
gathering in the natural enviroment surrouding their homes. This method of
harvesting from nature's provision is the oldest known subsistence stragy and has
been practised for at least the last two million years. It was, indeed, the only way to
obtain food until rudimentary farming and the domestication of wild animals were
introduced about 10,000 years ago.
Because hunter - gatherers have fared poorly in comparison with their
agricultural cousins, their numbers have dwindled, and they have been forced to
live in marginal environments, such as deserts and arctic wastelands. In higher
latitudes, the shorter growing seasons have restricted the availability of plant life.
Such conditions have caused a greater dependence on hunting, and on fishing along
the coasts and waterways. The abundance of vegetation in the lower latitudes of the
tropics, in the other hand, has provided a greater opportunity for gathering a variety
of plants. In short, the environmental differences have restricted the diet and have
limited possibilities for the development of subsistence societies.
Contemporary hunter - gatherers may help us understand our prehistoric
ancestors. We know from the observation of modem hunter - gatherers in both
Africa and Alaska that a society based on hunting and gathering must be very
mobile.
While
the entire community camps in a central
location,
a smaller party

harvests the
food
within
a reasonable distance
from
the camp. When the
food
in the
area
has become exhausted, the community moves on to
exploit
another site. We
also notice
seasonal
migration
patterns
evolving
for most hunter - gatherers, along
with
a
strict
division
of labor between the
sexes.
These patterns of behaviour may
be
similar
to those practised by mankind
during
the Paleolithic Period.

Question 61: The
word
"domestication" in the
first
paragraph mostly
means
*
A.
teaching animals to do a
particular
job
or
activity
in the home
B.
hatching and raising new
species
of
wild
animals in the home
C.
making
wild
animals used to
living
with
and
working
for humans
D.

adapting animals to suit a new
working
environment
Question 62:
According
to the
passage,
subsistence societies depend
mainly
on
A.
agricultural
products B. hunter - gatherers' tools
C.
nature's
provision
D.
farming
methods
Question 63: The
word
"marginal" in the second paragraph is closest in meaning
to
" "
A.
suburban B. abandoned C. forgotten D. disadvantaged
Question 64: In the
lower
latitudes of
the

tropics, hunter - gatherers
A.
have better
food
gathering
from
nature
B.
can free themselves
from
hunting
C.
harvest shorter
seasonal
crops 1
D.
live
along the
coasts
and waterways for
fishing
Question 65:
According
to the
passage,
studies of contemporary subsistence
societies can provide a
A.
broader
vision

of
prehistoric
natural environment
B.
deeper
insight
into
the dry - land
farming
C.
further understanding of modern subsistence societies
D.
further understanding of
prehistoric
times
Question 66: The
word
"conditions'"
in the second paragraph refers to
A.
tile
environments where it is not favorable for vegetation to grow
B.
the situations in
which
hunter - gatherers hardly
find
anything
to eat
C.

the places where plenty of animals and f1sh can be
found
D.
the situations in
which
hunter - gatherers can grow some crops
Question 67: A
tvpical
feature of
both
modern and prehistoric hunter - gatherers is
that
A.
they don't have a strong
sense
of
community
B.
they
live
in the forests for
all
their
life
C.
they don't have a healthy and balanced diet
D.
tlic_\n
change their
living

places
Question 68:
According
to the
passage,
which
of the
following
is NOT mentioned.'
A.
Hunting
or
fishing
develops where there are no or short
growing
seasons.
B.
The environmental differences produce no effect on subsistence societies.
C.
Harvesting
from
the natural environment had existed
long
before
farming
was taken up.
D.
The number of hunter - gatherers
decreases
where

farming
is convenient.
Question 69:
According
to the author, most contemporary and prehistoric hunter-
gatherers
share
A.
only
the way of
duty
division
B. some restricted
daily
rules
C.
some patterns of behaviour D. some methods of
production
Question 70:
Which
of
the
following
would
serve as the
best
title
of
the
passage?

A.
Hunter-gatherers:
Always
on the
Move
B.
Hunter-gatherers and Subsistence Societies
C.
A
Brief
History
of Subsistence Farming
D.
Evolution
of Humans' Farming Methods
Read
the following
passage
and mark the
letter
A, B, C or D on your answer
sheet to indicate the correct wordfor each
of
the blanks from
71
to
HO.
The
country is more
beautiful

than a
town
and (7!) to
live
in.
Many
people
think
so and go to the country for the summer holidays though they can't
live
there all the year round. (72) have a cottage (73) in a
village
(74)
they can go there whenever they can
find
the
time.
English
villages are not all (75) but in some ways they are not very
different
(76) each other. (77) every
village
has a church. The round
or
square
tower of
which
can be
seen
for many miles around. Surrounding the

church
is the (78) where people are buried.
The
village
green is a
wide
stretch of
grass
and
houses
or cottages are
built
round
it. Country
life
is now (79) comfortable and many villages have
water
brought through pipes
into
each home.
Most
villages are so close to some small towns (80) people can go
there to buy what they can't
find
in the
village
shops.
Question 71: A. peaceful
B.
pleasant

C.
pleasanter
D.
fairly
Question 72: A. Some
B.
Almost
C.
Most
D.
Others
Question 73: A. to
build
B.
building
C.
having
built
D.
built
Question 74: A. so that
B.
in order to
C.
so on
D.for
instance
Question 75: A.
like
B.

alike
C.
likely
D.
similar
Question 76: A. to
B.
from
C.
with
D.about
Question 77: A.
Most
B.
Most
of
C.
Almost
D.
Almost
of
Question 78: A. churchyard
B.
port
C.
path
D.
roundabout
Question 79: A.
fairly

B.
alike
C.
rarely
D.
hardly
Question 80: A. and
B.
that
C.
however
D.
nevertheless
DIEN
GIAI
DAP AN
Question I. Dap an dung la A. Since
Giai
thich
ve mat
ngiT
phap:
Ta
da biet sau Since la mot menh de dung de chi ly do.
Since + clause: Vi T~]
Cac phuong an C.
Because
of va D. On account of khong phii hop cau tnic ngir
phap. Ta da
biet

because
of/on
account of + N/N.P: vi
Phuong an B. Why: tai sac. khong logic y nghTa cua cau van da cho.
Question 2. Dap an dung la B. bankrupt: pha san
Giai
thich
ve mat y nghTa: Neu ho khong
than
trong khi sir dung von thi ho se
rai
vao tinh trcingphc'i
sari.
Cac
trucTng
hop khac khong logic ve mat y
nghTa:
-
A. poor (a): ngheo
-
C. penniless (a): khong mot xu dinh
tiii
-
D. broken (break - broke): lam va
Question 3. Dap an dung la A. patient enough
Giai
thich
ve mat ngu' phap: Diem ngir phap chinh cua cau hoi nay la each diing
Enough.
Ta da biet Enough ccS hai vj tri chinh: no dung ngay sau tinh tir va trang tir

nhung dung ngay truoc danh tir.
adj/
adv + enough: du
Y,-_,/
, enough + noun: du
Cac phuong an khac khong phii hop cau true ngu phap:
-
B. so patient. Ta c6 cau true:
[i ••••SO
+ adj/adv + that + clause: qua •den noi
-
D. too patient. Ta c6 cau tnic:
too + adj/adv for (s.o) + Vto inf: qua den noi khong the lam gi
-
Phuong an C. enough patient, sai hinh thiirc ngir phap.
Question 4. Dap an dung la C. get on well: hoa thuan
Giai
thich
ve mat y nghTa: Neu bo me
song
khong hoa thuan thi con cai se bat hanh.
Cac truong hop khac khong logic ve mat y nghTa:
-
get over: vugt qua
-
get off: thoat
khoi,
thoat ra, biroc xuong xe tau
-
get way: di nghi

Question 5. Dap an
diing
la C. unconscious: bat tinh
Giai
thich
ve mat y nghTa: Ngucri dan ong duoc dua den benh vien da bat tinh
trong
ba gia dong ho.
Cac truong hop khac khong logic ve mat y
nghTa:
-
' A. unfeeling (a): khong co cam xiic, v6 cam . '
.
B. indifferent (a): khong khac nhau
.
D. sensible (a):
hicu.
co
hoac
bieu 16 6c xet doan
Question 6. Dap an dung la B. neither
does
peter
Giai
thich
ve mat
ngiT
phap: Dung Neilher de dap lai y
CUNG
KHONG.

Co hai
van
de chung ta can luu v:
Mot
la: ve truoc phai co
hinh
thirc
hoac
y
phii
djnh.
Hai
la: Ve dap lai dung voi Neither khong bao g\b co
hinh
thirc phu
djnh.
Neither
+ trg dong tir/dong tir tobe + S.
Ta
khong dung So + tm dong tir/dong tir tobe + S de dap lai y
phii
djnh.
Question 7. Dap an
diing
la D. the most obvious
Gjai
thich
ve mat ngu phap: Ta can sir dung
hinh
thiirc

tinh
tir obvious a
viing
trong
ciia
cau van \ dong tir tobe (is) dang dugc
diing.
Hon nira, "obvious" la mot
tinh
tir dai ncn trong truong hop nay no phai tuan theo
hinh
thirc so
sanh
nhat cua
tinh
tir
diii.
< •
the most + long adj
Dich
nghTa:
.ifaii
loc la mot trong ninmg dgc diem rd
rang
nhdt de nhan dien
con
ngm'ri.
Question
H.
Dap an

diing
la A. discouraged (a) nan
long,
nan chi
Giai
thich
ve mat y nghTa: Anh ta thay nan chi khi anh ta rot ky thi Ian
thir
hai.
Cac phuong an khac khong
phii
hop voi y nghTa cau goc.
-
B. annoyed (v): lam phien '
-
C. undecided (a): khong quyet doan, do du
-
D. determined (v): quyet tam, xac
dinh
Question 9. Dap an
diing
la B. Each: moi, mot
Giai
thich
ve mat ngir phap: Ta da biet
Each of + the
/tinh
tir so huu + danh tir dem dirge so nhieu + dong tir so it
Cac phuong an con lai khong
phii

hop ve mat ngir phap:
-
A. Every + danh tir dem dugc so it + dong tir so it
-
C. Other + danh tir dem dugc so nhieu + dong tir so nhieu
-
D.
All
+ of + the/tinh tir so huu + danh tir dem dugc so nhieu + dong tir so nhieu
Question 10. Dap an
diing
la D.
if
that was
Giai
thich
ve mat ngu phap: Quan sat cau de cho, ta biet day la
hinh
thirc cau
tuong
tluiat
gian tiep (co dong tir tucmg
lluuTt
asked). Ta
loai
hai phuong an A. if
Is
this
va
/i.

if this is vi thi hien tai don khong ton tai trong
hinh
thirc cau tuong thuat
gian
tiep ma cu the la trong
truimg
hgp nay. Ta
loai
phuong an C. if was that vi
hinh
thirc cau tucrng thuat gian tiep khong co trucrng hgp dao dong tir len truoc
chii
ngCr
hoac
dai tir chi
dinh.
Question II. Dap an
diing
la D. had
left
Giai
thich
ve mat ngu phap: Cau
triic
ngu phap chu dao
ciia
cau van da cho la
By
the time. Ta da biet:

×